Sei sulla pagina 1di 111

EME 3142 MACHINE VIBRATIONS

Lecture Notes Compiled By:

Onesmus Mũtukũ Mũvengei


c May 2013

Students are reminded not to treat these lecture notes as a comprehensive and solely sufficient material
for their studies since the purpose of the notes is not meant to be a substitute for regularly attending
classes, reading relevant textbooks and recommended books. The notes are aimed at providing a quick
reference and a brief guidance for the students.
i

EME 3142 Machine Vibrations

Course Description

Vibration in machines: Types and sources. Transient vibrations: Discrete systems; damped
and undamped. Analysis of continuous systems; classical and finite element techniques; appli-
cations. Torsional vibration: Gear and shaft systems. Experimental vibration analysis:
Using Fast Fourier Transform (FFT) techniques. Vibration measurement.

Reference

1. Rao, S. S.(1986) Mechanical Vibrations, John Wiley & Sons, 5th Ed.

2. Benson H. Tongue (1996) Principles of Vibration, Oxford University press,

Topics Coverage
1. Introduction to vibration in machines
2. Vibration of single degree-of-freedom systems
3. Vibration of multiple degree-of-freedom systems
4. Transient vibrations
5. Vibration analysis of continuous systems using classical and finite element methods
6. Vibration measurement
Contents

Course Outline i

Table of Contents ii

1 Introduction to Vibrations in Machines 1

1.1 Causes of Vibrations . . . . . . . . . . . . . . . . . . . . . . . . . . . . . . . . . . 1

1.2 Characteristics of Vibration . . . . . . . . . . . . . . . . . . . . . . . . . . . . . . 3

1.2.1 Parts of a Vibrating System . . . . . . . . . . . . . . . . . . . . . . . . . . 3

1.2.2 Degree of Freedom (dof) of a Vibrating System . . . . . . . . . . . . . . . 4

1.2.3 Discrete and Continuous Systems . . . . . . . . . . . . . . . . . . . . . . . 6

1.2.4 Free and Forced Vibration . . . . . . . . . . . . . . . . . . . . . . . . . . . 6

1.2.5 Undamped and Damped Vibration . . . . . . . . . . . . . . . . . . . . . . 6

1.3 Vibration Analysis Procedure . . . . . . . . . . . . . . . . . . . . . . . . . . . . . 7

1.4 Spring Elements . . . . . . . . . . . . . . . . . . . . . . . . . . . . . . . . . . . . . 7

1.4.1 Springs in Parallel . . . . . . . . . . . . . . . . . . . . . . . . . . . . . . . 8

1.4.2 Springs in Series . . . . . . . . . . . . . . . . . . . . . . . . . . . . . . . . 9

1.5 Mass or Inertia Elements . . . . . . . . . . . . . . . . . . . . . . . . . . . . . . . . 10

1.6 Damping Elements (Damper) . . . . . . . . . . . . . . . . . . . . . . . . . . . . . 10

1.7 Harmonic Motion . . . . . . . . . . . . . . . . . . . . . . . . . . . . . . . . . . . . 11

1.7.1 Terminologies . . . . . . . . . . . . . . . . . . . . . . . . . . . . . . . . . . 12
CONTENTS iii

2 Single Degree of Freedom Systems 14

2.1 Free Vibrations . . . . . . . . . . . . . . . . . . . . . . . . . . . . . . . . . . . . . 14

2.1.1 Critical Damping Constant and the Damping Ratio . . . . . . . . . . . . . 15

2.1.2 Case 1: Undamped Vibrations - When ζ = 0 . . . . . . . . . . . . . . . . . 16

2.1.3 Case 2: Underdamped System - When ζ < 1 . . . . . . . . . . . . . . . . . 18

2.1.4 Case 3: Critically Damped System - When ζ = 1 . . . . . . . . . . . . . . 18

2.1.5 Case 4: Overdamped System - When ζ > 1 . . . . . . . . . . . . . . . . . . 19

2.1.6 Logarithmic Decrement . . . . . . . . . . . . . . . . . . . . . . . . . . . . . 19

2.2 Forced Vibrations . . . . . . . . . . . . . . . . . . . . . . . . . . . . . . . . . . . . 23

2.2.1 Undamped Forced Vibrations . . . . . . . . . . . . . . . . . . . . . . . . . 23

2.2.2 Damped Forced Vibrations . . . . . . . . . . . . . . . . . . . . . . . . . . . 25

3 Vibrations of Multi-Degree of Freedom Systems 36

3.1 Introduction . . . . . . . . . . . . . . . . . . . . . . . . . . . . . . . . . . . . . . . 36

3.2 Frequencies and Mode Shapes for Undamped Systems . . . . . . . . . . . . . . . . 36

3.2.1 Using Newton’s Second Law to Derive Equations of Motion . . . . . . . . . 37

3.2.2 Influence Coefficients . . . . . . . . . . . . . . . . . . . . . . . . . . . . . . 43

3.3 Numerical Methods of Finding the Natural Frequencies and Mode Shapes . . . . . 50

3.3.1 Matrix Iteration . . . . . . . . . . . . . . . . . . . . . . . . . . . . . . . . . 50

3.3.2 Holzer’s Method . . . . . . . . . . . . . . . . . . . . . . . . . . . . . . . . 54

4 Transient Vibrations 65

4.1 Introduction . . . . . . . . . . . . . . . . . . . . . . . . . . . . . . . . . . . . . . . 65

4.2 Impulse Excitations . . . . . . . . . . . . . . . . . . . . . . . . . . . . . . . . . . . 66

4.2.1 Impulse Response of Undamped Single Degree of Freedom System . . . . . 66

4.2.2 Impulse Response of Under-damped Single Degree of Freedom System . . . 67

4.3 Arbitrary Excitations . . . . . . . . . . . . . . . . . . . . . . . . . . . . . . . . . . 69

4.3.1 Convolution Integral . . . . . . . . . . . . . . . . . . . . . . . . . . . . . . 69

4.4 Description of a Transient Response . . . . . . . . . . . . . . . . . . . . . . . . . . 73


CONTENTS iv

5 Vibration of Continuous Systems 76

5.1 Introduction . . . . . . . . . . . . . . . . . . . . . . . . . . . . . . . . . . . . . . . 76

5.2 Vibration Analysis of Continuous Systems Using Analytical Method . . . . . . . . 76

5.2.1 Longitudinal Vibration of Bars . . . . . . . . . . . . . . . . . . . . . . . . 76

5.2.2 Torsional Vibration of Rods . . . . . . . . . . . . . . . . . . . . . . . . . . 80

5.2.3 Lateral Vibrations of Beams . . . . . . . . . . . . . . . . . . . . . . . . . . 83

5.3 Vibration Analysis of Continuous Systems Using Finite Element Method . . . . . 88

5.3.1 Introduction . . . . . . . . . . . . . . . . . . . . . . . . . . . . . . . . . . . 88

5.3.2 Mass and Stiffness Matrices for a Finite Element . . . . . . . . . . . . . . 89

5.3.3 Equation of Motion of the Complete System of Finite Elements . . . . . . 92

5.3.4 Incorporation of Boundary Conditions . . . . . . . . . . . . . . . . . . . . 93

6 Vibration Measurement 97

6.1 Vibration Measurement Scheme . . . . . . . . . . . . . . . . . . . . . . . . . . . . 97

6.2 Transducers . . . . . . . . . . . . . . . . . . . . . . . . . . . . . . . . . . . . . . . 98

6.2.1 Variable Resistance Transducers . . . . . . . . . . . . . . . . . . . . . . . . 99

6.2.2 Piezoelectric Transducers . . . . . . . . . . . . . . . . . . . . . . . . . . . . 99

6.2.3 Linear Variable Differential Transformer (LVDT) Transducer . . . . . . . . 100

6.3 Vibration Pickups . . . . . . . . . . . . . . . . . . . . . . . . . . . . . . . . . . . . 100

6.3.1 Vibrometer . . . . . . . . . . . . . . . . . . . . . . . . . . . . . . . . . . . 102

6.3.2 Accelerometers . . . . . . . . . . . . . . . . . . . . . . . . . . . . . . . . . 102

6.4 Signal Analysis . . . . . . . . . . . . . . . . . . . . . . . . . . . . . . . . . . . . . 105

6.4.1 Use of Fast Fourier Transform in Signal Analysis . . . . . . . . . . . . . . 106


Chapter 1

Introduction to Vibrations in Machines

Vibration in a machine is simply defined as the cyclic or oscillating motion of a machine or machine
component from its position of equilibrium.

The presence of vibration in a machine can lead to excessive wear of bearings, formation of cracks,
loosening of fasteners, structural and mechanical failures, frequent and costly maintenance of
machines, electronic malfunctions through fracture of solder joints, and abrasion of insulation
around electric conductors causing shorts. The occupational exposure of humans to vibration
leads to pain, discomfort, and reduced efficiency.

Vibration is not always a problem. In some tasks, vibration is essential. Machines such as
oscillating sanders and vibratory tumblers use vibration to remove materials and finish surfaces.
Vibratory feeders use vibration to move materials. In construction, vibrators are used to help
concrete settle into forms and compact fill materials. Vibratory rollers help compress asphalt used
in highway paving.

1.1 Causes of Vibrations

Vibration in machines is caused by the forces generated within the machine. These forces may:

(a) Change in direction with time, such as the force generated by a rotating unbalance.

(b) Change in amplitude or intensity with time, such as the unbalanced magnetic forces generated
in an induction motor due to unequal air gap between the motor armature and stator (field).

(c) Result in friction between rotating and stationary machine components in much the same way
that friction from a rosined bow causes a violin string to vibrate.

(d) Cause impacts, such as gear tooth contacts or the impacts generated by the rolling elements
of a bearing passing over flaws in the bearing raceways.
Vibrations: 1.1 Causes of Vibrations 2

(e) Cause randomly generated forces such as flow turbulence in fluid-handling devices such as
fans, blowers and pumps; or combustion turbulence in gas turbines or boilers.

Some of the most common machinery problems that cause vibration include:

(a) Misalignment of couplings, bearings and gears: Vibration can result when machine shafts are
out of line. Angular misalignment occurs when the axes of the driver and driven shaft (for
example, shafts of the motor and pump) are not parallel. When the axes are parallel but
not exactly aligned, the condition is known as parallel misalignment. Misalignment may be
caused during assembly or develop over time, due to thermal expansion, components shifting
or improper reassembly after maintenance. The resulting vibration may be radial or axial (in
line with the axis of the machine) or both.

(b) Imbalance in a rotating component: A “heavy spot” in a rotating component will cause
vibration when the unbalanced weight rotates around the machine’s axis, creating a centrifugal
force. Imbalance could be caused by manufacturing defects (machining errors, casting flaws) or
maintenance issues (deformed or dirty fan blades, missing balance weights). As machine speed
increases, the effects of imbalance become greater. Imbalance can severely reduce bearing life
as well as cause undue machine vibration.

(c) Looseness: Vibration that might otherwise go unnoticed may become obvious and destructive
if the component that is vibrating has loose bearings or is loosely attached to its mounts.
Such looseness may or may not be caused by the underlying vibrations. Whatever its cause,
looseness can allow any vibration present to cause damage, such as further bearing wear, wear
and fatigue in equipment mounts and other components.

(d) Wear: As components such as ball or roller bearings, drive belts or gears become worn, they
may cause vibration. When a roller bearing race becomes pitted, for instance, the bearing
rollers will cause a vibration each time they travel over the damaged area. A gear tooth that
is heavily chipped or worn, or a drive belt that is breaking down, can also produce vibration.

(e) Rubbing

(f) Aerodynamic/hydraulic problems in fans, blowers and pumps

(g) Electrical problems (unbalance magnetic forces) in motors

(h) Resonance: If the frequency of the exciting force coincides with one of the natural frequencies
of the system, a condition known as resonance occurs, and the system undergoes dangerously
large oscillations. Failures of such structures as buildings, bridges, turbines, and airplane
wings have been associated with the occurrence of resonance.
Vibrations: 1.2 Characteristics of Vibration 3

1.2 Characteristics of Vibration

When an elastic body such as a spring, beam or a shaft is displaced from its equilibrium position
by application of an external force and then released, it executes a vibratory motion. This is due
to the continuous conversion of the stored elastic energy to kinetic energy and vice vasa.

Any motion that repeats itself after an interval of time is called vibration or oscillation. Example:
the swinging of a pendulum, the motion of a plucked string etc.

The theory of vibration deals with the study of oscillatory motions of bodies and the forces
associated with them.

1.2.1 Parts of a Vibrating System

Whenever vibration occurs, there are actually four (4) elements involved that determine the char-
acteristics of the vibration. These elements as illustrated in Figure 1.1 are:

1. Spring or elasticity: This stores the potential energy.

2. Mass or inertia: This stores the kinetic energy.

3. Damper: This is the means in which energy is gradually lost.

4. The exciting force, such as imbalance or misalignment.

Figure 1.1: Elementary parts of a vibrating system

The vibration of a undamped system involves the transfer of its potential energy to kinetic energy
and of kinetic energy to potential energy, alternately. If there is a damper in the vibrating system,
some energy is dissipated in each cycle of vibration and should be replaced by an external source
if a steady state of vibration is to be maintained.

To appreciate this, consider the vibration of a simple pendulum shown in Figure 1.2;
Vibrations: 1.2 Characteristics of Vibration 4

Figure 1.2:

At position 1 before release,

• the ball has zero velocity and hence no kinetic energy

• the ball has a potential energy equal to mgl(1 − cosθ) with respect to datum position 2.

Once the ball is release, the ball starts to swing to the left due to the torque of the gravitational
force mg and by the time it reaches position 2, all of its potential energy will be converted into
kinetic energy. Hence the bob will not stop in position 2 but will continue to swing to position 3.

However, as the ball passes the mean position 2, a counterclockwise torque due to gravity starts
acting on the bob and causes the bob to decelerate. The velocity of the bob reduces to zero at
the left extreme position 3. By this time, all the kinetic energy of the bob will be converted to
potential energy.

Again due to the gravity torque, the bob continues to attain a counterclockwise velocity. Hence
the bob starts swinging back with progressively increasing velocity and passes the mean position
again. This process keeps repeating, and the pendulum will have oscillatory motion.

However, in practice, the magnitude of oscillation gradually decreases and the pendulum ultimately
stops due to the resistance (damping) offered by the surrounding medium (air) and the friction
at joint O. This means that some energy is dissipated in each cycle of vibration due to damping.

1.2.2 Degree of Freedom (dof ) of a Vibrating System

The minimum number of independent coordinates required to determine completely the position
of all parts of a system at any instant of time defines the degree of freedom of the system.

For instance the simple pendulum shown in Figure 1.2 is a single-degree-of-freedom system since
the motion of the simple pendulum is fully defined in terms of the angle θ. Other single-degree-
of-freedom systems are shown in Figure 1.3;
Vibrations: 1.2 Characteristics of Vibration 5

Figure 1.3:

Figures 1.4 and 1.5 show systems which can be described respectively as two dof and three dof
systems.

Figure 1.4: Two degree of freedom systems

Figure 1.5: Three degree of freedom systems

The coordinates necessary to describe the motion of a system constitute a set of generalized
coordinates. These are usually denoted as q1 , q2 , q3 ....qn where n is the number of dof of the
system.
Vibrations: 1.2 Characteristics of Vibration 6

1.2.3 Discrete and Continuous Systems

A Discrete (Lumped) system is a system which can be described using a finite number of degree
of freedom, such as the ones represented in Figures 1.2, 1.4 and 1.5.

A Continuous (Distributed) system is a system which can be described using a finite number of
degree of freedom, such as a cantilever beam shown in Figure 1.6.

Figure 1.6: A cantilever beam (an infinite-number-of-degrees-of-freedom system).

Most structural and machine systems have deformable (elastic) members and therefore have an
infinite number of degrees of freedom.

Most of the time, continuous systems are approximated as discrete systems, and solutions are
obtained in a simpler manner. Although treatment of a system as continuous gives exact results.

1.2.4 Free and Forced Vibration

If a system, after an initial disturbance, is left to vibrate on its own, the ensuing vibration is known
as free vibration. No external force acts on the system. The motion is maintained by gravitational
or elastic restoring forces, such as the swinging motion of a pendulum or the vibration of an elastic
rod.

If after an initial disturbance the system is subjected to an external force (often, a repeating type of
force), the resulting vibration is known as forced vibration. The oscillation that arises in machines
such as diesel engines is an example of forced vibration. If the frequency of the external force
coincides with one of the natural frequencies of the system, a condition known as resonance occurs,
and the system undergoes dangerously large oscillations. Failures of such structures as buildings,
bridges, turbines, and airplane wings have been associated with the occurrence of resonance.

1.2.5 Undamped and Damped Vibration

If no energy is lost or dissipated due to friction or other resistance during oscillation, the vibration
is known as undamped vibration. This energy dissipation leads to reduction of the amplitude after
every cycle of vibration.

If any energy is lost or dissipated due to friction or other resistance during oscillation, the vibration
is called damped vibration. In many physical systems, the amount of damping is so small such that
Vibrations: 1.3 Vibration Analysis Procedure 7

it can be neglected for most engineering purposes. However, consideration of damping becomes
extremely important in analyzing vibratory systems near resonance.

1.3 Vibration Analysis Procedure

A vibratory problem is a dynamic one for which the variables such as the excitations (inputs)
and responses (outputs) are time dependent. Thus the analysis of a vibrating system usually
involves mathematical modeling, derivation of the governing equations, solution of the equations,
and interpretation of the results.

1. Mathematical modeling: This is the first step which involves representing all the important
features of the system in a way which will easily ensure that the the mathematical equations
governing the system’s behavior are derived.

2. Derivation of the governing equations: Once the mathematical model is available, the prin-
ciples of dynamics are then used to derive the equations that describe the vibration of the
system.

3. Solution of the governing equations: This involves using the analytical and numerical pro-
cedures to solve the derived equations so as to get the output of the vibrating system.

4. Interpretation of the Results: The results of the vibrating system are interpreted with a
clear view of the purpose of the analysis and the possible design implications of the results.

1.4 Spring Elements

A spring is a type of mechanical element, which in most applications is assumed to have negligible
mass and damping. The most common type of spring is the helical-coil spring used in retractable
pens staplers, and suspensions of vehicles.

Any elastic or deformable body or member, such as a cable, bar, beam, shaft or plate, can be
considered as a spring.

A spring is said to be linear if the elongation or reduction in length x (deflection) is related to the
applied force F as
F = kx (1.1)
where F is applied force and x is the component’s change in length from its original length. k is
a constant, known as the spring constant or spring stiffness or spring rate. The spring constant
k is always positive and denotes the force (either tensile of compressive) required to cause a unit
deflection in the spring.
Vibrations: 1.4 Spring Elements 8

The work done (U) in deforming a spring is stored as strain or potential energy in the spring, and
it is given by:
1
U = kx2 (1.2)
2

1.4.1 Springs in Parallel

The springs of Figure 1.7 are said to be in parallel.

Figure 1.7:

If the block is subject to an arbitrary displacement x, the change in length of each spring in the
parallel combination is x. The free body diagram of Figure 1.8 shows that the total force acting
on the block is

Figure 1.8:

n
X
F = k1 x + k2 x + k3 x + . . . + kn x = ki x (1.3)
i=1

If keq is the equivalent spring constant of the combination of the springs in parallel, then for the
same static deflection x the force acting on the block is equal to the force of Equation (1.3). That
is:

n
X
F = keq x = ki x (1.4)
i=1

Therefore:
Vibrations: 1.4 Spring Elements 9

n
X
keq = ki (1.5)
i=1

1.4.2 Springs in Series

The springs in the system of Figure 1.9 are said to be in series.

Figure 1.9:

Let x be the displacement of the block of Figure 1.9 at an arbitrary instant. Let xi be the change
in length of the ith spring from the fixed support, then the total deflection of the springs in series
is:

n
X
x = x1 + x2 + x3 + . . . + xn = xi (1.6)
i=1

If each spring is assumed massless, then the force developed at each end of the spring has the
same magnitude but opposite in direction, as shown in Figure 1.10. Thus the force is the same in
each spring and is given as:

F = k1 x1 = k2 x2 = k3 x3 = . . . = kn xn (1.7)

Figure 1.10:

If keq is the equivalent spring constant of the combination of the springs in series, then for the
same static deflection x the force acting on the block is equal to the force of Equation (1.7). That
is:

F = k1 x1 = k2 x2 = k3 x3 = . . . = kn xn = keq x (1.8)
which can be shown to reduce to:
n
1 1 1 1 1 X 1
= + + + ... = (1.9)
keq k1 k2 k3 kn k
i=1 i
Vibrations: 1.5 Mass or Inertia Elements 10

In certain applications, springs are connected to rigid components such as pulleys, levers, and
gears. In such cases, an equivalent spring constant can be found using energy equivalence.

1.5 Mass or Inertia Elements

The mass or inertia element is assumed to be a rigid body; it can gain or lose kinetic energy
whenever the velocity of the body changes.

In many practical applications, several masses appear in combination, either in parallel or in


series. For instance, consider a multistory building subjected to an earthquake. Assuming that
the mass of the frame is negligible compared to the masses of the floors, the building can be
modeled as a multi-degree-of-freedom system, as shown in Figure 1.11. The masses at the various
floor levels represent the mass elements, and the elasticities of the vertical members denote the
spring elements.

Figure 1.11:

For a simple analysis, we can replace these masses by a single equivalent mass.

1.6 Damping Elements (Damper)

As seen earlier, there can be energy dissipation in form of heat or sound during vibration due to
friction or other resistance.

Although the amount of energy converted into heat or sound is relatively small, the consideration
of damping becomes important for an accurate prediction of the vibration response of a system.
Vibrations: 1.7 Harmonic Motion 11

A damper is assumed to have neither mass nor elasticity, and damping force exists only if there
is relative velocity between the two ends of the damper.

Damping is modeled as one or more of the following types:

1. Viscous Damping: This is the most commonly used damping mechanism in vibration anal-
ysis. When mechanical systems vibrate in a fluid medium such as air, gas, water, or oil, the
resistance offered by the fluid to the moving body causes energy to be dissipated. In viscous
damping, the damping force is proportional to the velocity of the vibrating body. Hence a
linear viscous damping component has a force-velocity relation of the form:

F = cv (1.10)

where c is the damping coefficient of dimensions kg/s.


Typical examples of viscous damping include; fluid film between sliding surfaces, fluid flow
around a piston in a cylinder, fluid flow through an orifice, and fluid film around a journal
in a bearing.
A dashpot is a mechanical device that adds viscous damping to a mechanical system and is
represented as symbolically as shown in Figure 1.12.

Figure 1.12:

2. Coulomb or Dry-Friction Damping: The damping force is constant in magnitude but opposite
in direction to that of the motion of the vibrating body. It is caused by friction between
rubbing surfaces that either are dry or have insufficient lubrication.

3. Material or Solid or Hysteretic Damping: When a material is deformed, energy is absorbed


and dissipated by the material due to friction between the internal planes, which slip or slide
as the deformations take place.

1.7 Harmonic Motion

If motion is repeated after equal intervals of time, it is called periodic motion. The simplest type
of periodic motion is harmonic motion.

When the acceleration is proportional to the displacement and directed toward the mean position,
the motion is referred to as simple harmonic motion.
Vibrations: 1.7 Harmonic Motion 12

~ of magnitude A
Harmonic motion can be represented conveniently by means of a vector OP
rotating at a constant angular velocity ω as shown in Figure 1.13

Figure 1.13:

1.7.1 Terminologies

The following definitions and terminology are useful in dealing with harmonic motion and other
periodic functions.

(a) Period of vibration or time period: This is the time interval after which motion repeats itself.
It is expressed in seconds. In other words, it is time taken to complete one cycle of motion.
From Figure 1.13, it is equal to the time required for the vector OP ~ to rotate through an
angle of 2π,and hence;

τ= (1.11)
ω
where ω is called the circular frequency.

(b) Amplitude: This is the maximum displacement of the vibrating body from its equilibrium
position.
Vibrations: 1.7 Harmonic Motion 13

(c) Cycle: This is the motion completed during one time period.

(d) Frequency of oscillation: Number of cycles executed in one second (cycles per second). In SI
units frequency is expressed hertz (Hz).
1 ω
f= = (1.12)
τ 2π

(e) Natural frequency. If a system, after an initial disturbance, is left to vibrate on its own, the
frequency with which it oscillates without external forces is known as its natural frequency.
As will be seen later, a vibratory system having n degrees of freedom will have, in general, n
distinct natural frequencies of vibration.
Chapter 2

Single Degree of Freedom Systems

2.1 Free Vibrations

Consider a mass-spring system with a viscous damper represented as shown in Figure 2.1

Figure 2.1:

Applying the dynamic equilibrium on the mass and noting that at static equilibrium W = kδst :
X
↓+ Fy = 0
−mẍ + W − k(x + δst ) − cẋ = 0
−mẍ + W − kx − kδst − cẋ = 0
mẍ + cẋ + kx = 0 (2.1)

This is a second order ode which can be solved by first assuming a solution of the form:

x = Cest (2.2)
Vibrations: 2.1 Free Vibrations 15

where s and C are constants to be determined. The first and second time derivatives of Equation
2.2 are:

ẋ = sCest
ẍ = s2 Cest

which when substituted in Equation 2.1 leads to:

ms2 Cest + csCest + kCest = 0


ms2 + cs + k = 0 (2.3)

Equation 2.3 is a quadratic equation whose roots are:



−c ± c2 − 4mk
s1,2 =
2m
r
c c 2 k
= − ± − (2.4)
2m 2m m

These two roots give two solutions to Equation 2.2 which when combined give the general solution
as:

x = C 1 e s1 t + C 2 e s2 t (2.5)

where C1 and C2 are arbitrary constants to be determined from the initial conditions of the system.

2.1.1 Critical Damping Constant and the Damping Ratio

The critical damping is defined as the value of the damping constant cc for which the radical in
Equation 2.4 becomes zero, that is;

−c ± c2 − 4mk
s1,2 =
r 2m
cc  2 k
− = 0
2m m r
k
cc = 2m = 2mωn (2.6)
m

For any damped system, the damping ratio ζ is defined as the ratio of the damping constant c to
the critical damping constant cc , that is:
c
ζ = (2.7)
cc

We can write:
c c cc
= × = ζωn (2.8)
2m cc 2m
Vibrations: 2.1 Free Vibrations 16

Therefore:
p
s1,2 = −ζωn ± ζ 2 ωn2 − ωn2
p
= (−ζ ± ζ 2 − 1)ωn (2.9)

Thus Equation 2.5 becomes:


√ √
ζ 2 −1)ωn t ζ 2 −1)ωn t
x = C1 e(−ζ+ + C2 e(−ζ− (2.10)

Depending on the damping coefficient, the mass and the stiffness, the value of ζ can take several
values which gives out four cases. These cases include:

2.1.2 Case 1: Undamped Vibrations - When ζ = 0

When ζ = 0, solution to Equation 2.10 becomes:

x = C1 eiωn t + C2 e−iωn t (2.11)

which is a solution to a free undamped vibration for a single degree of freedom system.

The general solution for Equation 2.11 can be shown to be:

x = A sin ωn t + B cos ωn t (2.12)

where A and B represent constants of integration. The velocity and acceleration of the block are
determined by taking successive time derivatives, yielding:

ẋ = Aωn cos ωn t − Bωn sin ωn t (2.13)


ẍ = −Aωn2 sin ωn t − Bωn2 cos ωn t (2.14)

Constants A and B can be determined using the initial conditions of the problem which are
represented as:
x(0) = x0 and ẋ(0) = ẋ0 (2.15)
Using these initial conditions to calculate constants A and B, the solution in Equation 2.12 be-
comes:
ẋ0
x = x0 cos ωn t + sin ωn t (2.16)
ωn

Equation 2.12 can be expressed in terms of simple sinusoidal motion. To show this, let:

A = C cos φ and B = C sin φ (2.17)

where C and φ are new constants to be determined instead of A and B. Equation 2.12 now
becomes:

x = C cos φ sin ωn t + C sin φ cos ωn t (2.18)


Vibrations: 2.1 Free Vibrations 17

Figure 2.2:

Since sin(θ + φ) = sin θ cos φ + cos θ sin φ, then Equation 2.18 becomes:

x = C sin(ωn t + φ) (2.19)

If Equation 2.19 is used to plot x versus ωn t, the graph of shape shown in Figure 2.2:

The angle φ is called the phase angle since it represents the amount by which the curve is displaced
from the origin when t=0s. This phase angle is given by:
 
−1 B
φ = tan (2.20)
A

The amplitude of the curve is equal to constant C which can be expressed in terms of constants
A and B as:

C = A2 + B 2 (2.21)

Using the initial conditions given in Equation 2.15, the phase angle and the amplitude given in
become:
 
−1 ω n x0
φ = tan (2.22)
ẋ0
s  2
2 ẋ0
C = x0 + (2.23)
ωn

If τ is the time in seconds taken for the sine curve shown in Figure 2.2 representing the harmonic
motion of the free undamped vibration of a mass-spring system to complete one cycle, (that is,
time period=τ s) then:

2π = ωn τ

τ = (2.24)
ωn
Vibrations: 2.1 Free Vibrations 18

Therefore the frequency of the vibrations is:


1
f =
τ
ωn
=
2π r
1 k
= (2.25)
2π m

When a body or a system of connected bodies is given an initial displacement from its equilibrium
position and released, it will vibrate with the natural frequency ωn . If the system has a single
degree of freedom, then the vibrating motion will have the same characteristics as the simple
harmonic motion of the block and spring just presented.

2.1.3 Case 2: Underdamped System - When ζ < 1

When ζ < 1, the solution of the roots in Equation 2.9 is complex conjugate:
p
s1,2 = (−ζ ± i 1 − ζ 2 )ωn (2.26)
Therefore the solution of the vibrations is:
√ √
2 2
x = C1 e(−ζ+i 1−ζ )ωn t + C2 e(−ζ−i 1−ζ )ωn t
 p p 
= e−ζωn t A cos( 1 − ζ 2 ωn t) + B sin( 1 − ζ 2 ωn t)
 
−ζωn t
= e A sin ωd t + B cos ωd t (2.27)
p
where ωd = 1 − ζ 2 ωn is the frequency of the damped vibrations. Constants A and B can be
found using the initial conditions.

Equation 2.27 can be shown to be an harmonic motion whose amplitude decays exponentially due
to occurrence of the factor e−ζωn t . This is shown in Figure 2.3.

It is clear that ωd < ωn , and the decrease in ωd increases with the increasing amount of the
damping.

2.1.4 Case 3: Critically Damped System - When ζ = 1

When ζ = 1, the solution roots s1 and s2 are the same, that is,

s1,2 = (−1 ± 12 − 1)ωn = −ωn (2.28)
Then the general solution of the vibrations is:
x = C1 eωn t + C2 eωn t
= (C1 + C2 )eωn t = Aeωn t (2.29)
As t → ∞ then eωn t → 0, hence the motion will eventually diminish to zero as shown in Figure
2.3. Such a motion is said to be aperiodic.
Vibrations: 2.1 Free Vibrations 19

Figure 2.3:

2.1.5 Case 4: Overdamped System - When ζ > 1

When ζ > 1, the roots are all negative and represented as:
p
s1 = (−ζ + ζ 2 − 1)ωn
p
s2 = (−ζ − ζ 2 − 1)ωn

Then the general solution of the vibrations is:


√ √
2 2
x = C1 e(−ζ+ ζ −1)ωn t + C2 e(−ζ− ζ −1)ωn t (2.30)

Both s1 and s2 are negative so that the solution of x in Equation 2.30 is a sm of a vanishing
exponential term. The motion is non-oscillatory and when the mass is slowly disturbed it will
slowly return to its equilibrium position. The damping is described as heavy and the motion is
also aperiodic. The the motion will exponentially diminish to zero with time as shown in Figure
2.3.

NOTE: Other than the the undamped vibration, Case 2 of the underdamped vibration is the most
practical one and is very important in the study of mechanical vibrations since it also leads to an
oscillatory motion.

2.1.6 Logarithmic Decrement

The logarithmic decrement represents the rate at which the amplitude of a free-underdamped
vibration decreases. It is defined as the natural logarithm of the ratio of any two successive
amplitudes.

Consider Figure 2.3 showing the underdamped vibration.


Vibrations: 2.1 Free Vibrations 20

Figure 2.4:

Equation 2.27 for underdamped system can be written in sinusoidal form as:

x = Xe−ζωn t sin(ωd t + φ)

Where:
p
ωd = ωn 1 − ζ2
B
φ = phase angle = tan−1
√ A
X = A2 + B 2

Let t1 and t2 denote the times corresponding to two consecutive amplitudes (or displacements)
on one side of the mean position measured one cycle. Therefore, the ratio of the consecutive
amplitudes is:

x1 Xe−ζωn t1 sin(ωd t1 + φ)
=
x2 Xe−ζωn t2 sin(ωd t2 + φ)

If the time period of the underdamped vibrations is τd = ωd
, then:


t2 = t1 + τd = t1 +
ωd
Therefore:
 2π 
sin(ωd t2 + φ) = sin ωd (t1 + )+φ
ωd
= sin(2π + ωd t1 + φ)
= sin(ωd t1 + φ)

This leads to:


x1 e−ζωn t1
= −ζωn (t1 +τ )
x2 e d

e−ζωn t1
= −ζωn t1 −ζωn τ = eζωn τd (2.31)
e .e d
Vibrations: 2.1 Free Vibrations 21


Logarithmic decrement δ is now given as: τd = ωd
, then:
x1
δ = ln( ) = ln(eζωn τd ) = ζωn τd (2.32)
x2
2π ζωn 2π
= ζωn = p
ωd ωn 1 − ζ 2
2πζ
= p (2.33)
1 − ζ2
For small damping ζ <<< 1 and hence the logarithmic decrement becomes δ = 2πζ.

Once the logarithmic decrement is obtained, the damping ratio ζ and the damping constant can
be found as follows:
2πζ
δ = p
1 − ζ2
4π 2 ζ 2
δ2 =
1 − ζ2
δ
ζ = √ (2.34)
4π + δ 2
2

and:
c c
ζ = =
cc 2mωn
c = 2mωn ζ (2.35)

The logarithmic ratio can also be found by measuring two amplitudes (or displacements) separated
by any number of complete cycles. If x1 and xm+1 denote the amplitudes (or displacements)
corresponding to t1 and tm+1 = t1 + mτd , where m is an integer denoting the number of cycles
separating the x1 and xm+1 , then:
x1 x1 x2 x3 xm
= . . ....... (2.36)
xm+1 x2 x3 x4 xm+1

From Equation 2.31, any two consecutive displacements satisfies this condition:
xj
= eζωn τd
xj+1
Therefore:
x1
= e(ζωn τd )m
xm+1
 x   
1
ln = ln e(ζωn τd )m = ζωn τd m
xm+1
(2.37)

But from Equation 2.32, δ = ζωn τd , hence:


x1
= δm
xm+1
1  x1 
δ = ln (2.38)
m xm+1
Vibrations: 2.1 Free Vibrations 22

Example 2.1.1. An underdamped shock absorber is to be designed for a motorcycle of mass 200 kg
as shown in Figure 2.5(a). When the shock absorber is subjected to an initial vertical velocity due
to a road bump, the resulting displacement-time curve is to be as indicated in Figure 2.5(b). Find
the necessary stiffness and damping constants of the shock absorber if the period of the damped
vibration is to be 2s and after one complete oscillation, the amplitude of the vibration decreases to
one-sixteenth of the initial amplitude value.

Figure 2.5:

Solution

The logarithmic decrement of the vibration is:


x1 x1
δ = ln = ln 1 = ln(16) = 2.7726
x2 x
16 1

Also logarithmic decrement has been shown to be:


2πζ
δ = p
1 − ζ2

from which the damping ratio can be shown to be:


δ 2.7726
ζ = √ =√ = 0.4037
4π 2 + δ 2 4π 2 + 2.77262

The periodic time of the damped vibrations is τd = 2s, then the frequency of the damped vibrations
becomes:
2π p
ωd = = ωn 1 − ζ 2
τd
2π 2π
ωn = p = √ = 3.4338rad/s
τd 1 − ζ 2 2 1 − 0.40372
Vibrations: 2.2 Forced Vibrations 23

The stiffness of the spring is:

k = mωn2 = 200 × 3.43382 = 2358.2652N/m

The damping constant of the damper is:

c = 2mωn ζ = 2(200)(3.4338) × 0.4037 = 554.4981N − s/m

2.2 Forced Vibrations

2.2.1 Undamped Forced Vibrations

Undamped forced vibration is considered to be one of the most important types of vibrating motion
in engineering. Its principles can be used to describe the motion of many types of machines and
structure.

If F (t) = Fo cos ωt excites mass m of an undamped system, the equation of motion can be shown
to be:

mẍ + kx = Fo cos ωt (2.39)

Equation 2.39, is a nonhomogeneous second order ordinary differential equation with homogeneous
and particular solutions.

The homogeneous solution to the equation is:

xh = A sin ωn t + B cos ωn t (2.40)

Since the exciting force F(t) is harmonic, the particular solution xp is also harmonic and has the
same frequency ω. We assume a solution of the form:

xp = X cos ωt

where X is the maximum amplitude of xp . The first and second time derivative of the particular
solution is:

ẋp = −ωX sin ωt


ẍp = −ω 2 X cos ωt
Vibrations: 2.2 Forced Vibrations 24

which when substituted in Equation 2.39 leads to:

− mω 2 X cos ωt + kX cos ωt = Fo cos ωt


X cos ωt (k − mω 2 ) = Fo cos ωt
Fo
Fo k
X = =
k − mω 2 1− m
k
ω2
δst
=  2 (2.41)
1 − ωωn

δst denotes deflection of the mass under a force Fo and is called static deflection because Fo is
constant (static) force.

The quantity:
X 1
=  2
δst ω
1− ωn

is called the magnification factor, amplification factor or amplitude ratio. It represents the ratio
of the dynamic to the static amplitudes of the motion.

The general solution is:

x = x h + xp
Fo
= A sin ωn t + B cos ωn t + cos ωt (2.42)
k − mω 2
Using the initial conditions x(t = 0) = x0 and ẋ(t = 0) = ẋ0 , we get constants A and B as:

Fo
B = x0 −
k − mω 2
ẋ0
A =
ωn
Then the solution becomes:
ẋ0  Fo  Fo
x = sin ωn t + x0 − cos ω n t + cos ωt (2.43)
ωn k − mω 2 k − mω 2

At resonance, the forcing frequency equals the natural frequency of the vibrations, that is, ωn = ω
and ωωn = 1. In this case the amplitude X is infinite as per Equation 2.41.

The solution in Equation 2.43 can also be written as:


ẋ0 Fo  
x = sin ωn t + x0 cos ωn t + cos ωt − cos ω n t
ωn k − mω 2
ẋ0  cos ωt − cos ω t 
n
= sin ωn t + x0 cos ωn t + δst  2 (2.44)
ωn
1 − ωωn
Vibrations: 2.2 Forced Vibrations 25

Using L’Hopitals rule at resonance to evaluate the limit of the last term of Equation 2.46 as
ω → ωn :
   
d
 cos ωt − cos ωn t  (cos ωt − cos ωn )t
lim   2  = lim  dω 
2
 
ω→ωn ω→ω d ω
1 − ωnω n

1 − ω2
n

(2.45)
" #
t sin ω
= lim
ω→ωn 2 ωω2
n

ωn t
= sin ωn t
2

Thus the response of the system at resonance becomes


ẋ0 δst ωn t
x = x0 cos ωn t + sin ωn t + sin ωn t (2.46)
ωn 2

At resonance as shown in Equation 2.46, the response of the system (x) increases indefinitely, and
the amplitude of the response increases linearly with time as shown in Figure 2.6.

Figure 2.6:

The design of any forced vibrating system should ensure that the forcing force frequency does not
equalize the natural frequency of the system in order to avoid the resonance.

2.2.2 Damped Forced Vibrations

Consider Figure 2.7 which shows a mass-spring-damper system whose mass is excited by an har-
monic force given as F (t) = Fo cos ωt

The equation of motion is:

mẍ + cẋ + kx = Fo cos ωt (2.47)


Vibrations: 2.2 Forced Vibrations 26

Figure 2.7:

If the system is underdamped, then the complementary (homogeneous) solution to Equation 2.47
which represents the transient vibrations is:

xh = eζωn t (A sin ωd t + B cos ωd t) (2.48)

where:
c c c
ζ = = = √
cc 2mωn 2 km
p
ωd = ωn 1 − ζ 2

Solution in Equation 2.48 can also be written as: 2.47 is

xh = Xe−ζωn t cos(ωd t − φ) (2.49)

where X and φ are the amplitude and the phase angle of the transient vibrations.

The particular solution to Equation 2.47 is also expected to be harmonic and can be assumed to
be of the form:

xp = Xo cos(ωt − φo ) (2.50)

where where Xo and φo are the amplitude and the phase angle of the steady-state vibrations due
to harmonic exciting force.

The first and second time derivatives of the particular solution are respectively:

ẋp = −ωXo sin(ωt − φo )


ẍp = −ω 2 Xo cos(ωt − φo )

which when substituted to Equation 2.47 gives:


h i
2
Xo (k − mω ) cos(ωt − φo ) − cω sin(ωt − φo ) = Fo cos ωt
Vibrations: 2.2 Forced Vibrations 27

Using the following trigonometric identities:

cos(ωt − φo ) = cos ωt cos φo + sin ωt sin φo


sin(ωt − φo ) = sin ωt cos φo − cos ωt sin φo

we get:
h i
2
Xo (k − mω )(cos ωt cos φo + sin ωt sin φo ) − cω(sin ωt cos φo − cos ωt sin φo ) = Fo cos ωt

Equating the coefficients of cos ωt and sin ωt on both sides of the above equation leads to the
following two equations:
h i
2
Xo (k − mω ) cos φo + cω sin φo = Fo (2.51)
h i
Xo (k − mω 2 ) sin φo − cω cos φo = 0 (2.52)

Squaring both sides of these equations and adding them leads to:
h i
2 2 2 2 2 2 2 2 2 2
Fo = Xo (k − mω ) (cos φo + sin sin φo ) + c ω (cos φo + sin sin φo )
h i
Fo2 = Xo2 (k − mω 2 )2 + c2 ω 2
Fo
Xo = h i 12 (2.53)
(k − mω 2 )2 + c2 ω 2

From Equation 2.52 and noting that Xo 6= 0, then:

(k − mω 2 ) sin φo − cω cos φo = 0
sin φo cω
= tan φo =
cos φo k − mω 2
h cω i
φo = tan−1 (2.54)
k − mω 2
Let:
ω
r =
ω
rn
k
ωn =
m
c c c
ζ = = = √
cc 2mωn 2 mk
c
= 2ζωn
m
Fo
δst =
k
Vibrations: 2.2 Forced Vibrations 28

Then:
Fo
Xo = h i 21
k2 − 2kmω 2 + m2 ω 4 + c2 ω 2
Fo
= h  i 12
2mkω 2 m2 ω 4 c2 ω 2
k2 1 − k2
+ k2
+ k2
Fo
k
= h 2
i
ω4 2 2
1 − 2 ωω2 + 4
ωn
+ (2ζωn )2 mk2ω 1
1
n

δst
= h i 21
ω2 2
(1 − 2)
ωn
+ (2ζ ωωn )2
δst
= h i 21 (2.55)
(1 − r2 )2 + (2ζr)2

The quantity:
Xo 1
= h i 21 (2.56)
δst 2 2 2
(1 − r ) + (2ζr)

is called the magnification factor, amplification factor or the amplitude ratio. It represents the
ratio of the dynamic to the static amplitudes of the motions.

Using Equations 2.53 and 2.54, the amplitude and the phase angle of the steady state vibrations
can be obtained. The general solution of a forced damped vibration now is:

x = xh + xp
x = Xe−ζωn t cos(ωd t − φ) + Xo cos(ωt − φo ) (2.57)

where constants X and φ which are different from the ones for free-damped vibration can be
obtained by applying the initial conditions x0 and ẋ0 to the Equation 2.57.

Example 2.2.1. A single cylinder vertical petrol engine of total mass 400kg is mounted on a steel
chassis frame and causes a vertical static deflection of 2.6667mm. The reciprocating parts of the
engine have a total mass of of 30kg and move through a vertical stroke of 200mm with simple
harmonic motion. A viscous damper is provided of a damping coefficient 1.5kN-s/m. Considering
that steady state state vibration of the engine is reached, determine:

(a) The amplitude of the forced vibrations when the driving shaft of the engine rotates at 500rpm.

(b) The speed of the driving shaft at which resonance will occur.

Solution
Vibrations: 2.2 Forced Vibrations 29

(a) The amplitude of the steady state vibrations is given as:


Fo
Xo = h i 21
(k − mω 2 )2 + c2 ω 2

The stiffness:
W 400 × 9.81
k = = −3
= 1.47 × 106 N/m
δst 2.6667 × 10

Radius of crank:
Lstroke 160
r = = = 80mm = 0.08m
2 2

Angular speed of the crank which is the exciting frequency:


2π × 500
ω = = 52.36rad/s
60

The centrifugal force due to the reciprocating parts is the static force of the exciting force:

Fo = mr an = mr ω 2 r = 30 × 52.362 × 0.08 = 6579.77N

Since steady state state is reached, then the amplitude of the forced vibration is:

Fo
Xo = h i 21
2 2 2
(k − mω ) + c ω 2

6579.77
= h i 21 = 0.01725m
6 2 2 2
(1.47 × 10 − 400 × 52.36 ) + 1500 × 52.362

(b) At resonance the natural frequency equalizes the exciting frequency:


r r
k 1.47 × 106
ω = ωn = = = 60.65rad/s
m 400

Therefore the speed of the driving shaft at resonance is:


60ω 60 × 60.65
N = = = 579.16rev/min
2π 2π
Vibrations: 2.2 Forced Vibrations 30

Example 2.2.2. Find the total response of a single degree of freedom system with m=10kg, c=20N-
s/m, k=4000N/m, x0 =0.01m and ẋ0 =0m/s when

(a) An external harmonic force F(t)=1000cos10t excites the system

(b) No exciting force on the system

Solution

(a) Total response for an harmonically excited vibrating system is given as:

x = Xe−ζωn t cos(ωd t − φ) + Xo cos(ωt − φo )

We need therefore to find all the constants in the equation above to get a general equation
of response of the system.
Fo
Xo = h i 21
2 2 2
(k − mω ) + c ω 2

100
= h i 21 = 0.03326
2 2 2
(4000 − 10 × 10 ) + 20 × 10 2
h cω i
φo = tan−1
k − mω 2
h 20 × 10 i
= tan−1 = 3.8140
4000 − 10 × 102

To find constants X and φ, we need to use the initial conditions.

x0 = X cos φ + Xo cos φo = 0.01


0.01 = X cos φ + 0.03326 cos 3.814
X cos φ = −0.023186 (2.58)

To get ẋ0 lets expand Equation 2.57, then get the first time derivative and substitute t=0.
This leads to

ẋ0 = −Xζωn cos φ + Xωd sin φ + Xo ω sin φo

where;
r r
k 4000
ωn = = = 20rad/s
m 10
c c 20
ζ = = = = 0.05
cc 2mωn 2 × 10 × 20
p √
ωd = ωn 1 − ζ 2 = 20 1 − 0.052 = 19.975rad/s
Xo = 0.03326
φo = 3.8140
Vibrations: 2.2 Forced Vibrations 31

Therefore:

0 = −X(0.05)(20) cos φ + 19.975X sin φ + 0.03326(10) sin 3.814


0 = −X cos φ + 19.975X sin φ + 0.02212

But from Equation 2.58 X cos φ = −0.023186, then:

0 = −(−0.023186) + 19.975X sin φ + 0.02212


X sin φ = −0.002268 (2.59)

Equations 2.58 and 2.59 can be solved simultaneously to get X and φ as

X = −0.023297m
φ = 5.5870

Hence the total response becomes:


π π
x = −0.023297e−0.05(20)t cos(19.975t − 5.587 × ) + 0.03326 cos(10t − 3.814 × )
180 180
= −0.023297e−t cos(19.975t − 0.0975) + 0.03326 cos(10t − 0.0665)

(b) The total response for free-underdamped vibrating system can be represented as:

x = Xe−ζωn t cos(ωd t − φ) (2.60)

We need therefore to find X and φ using the initial conditions x0 = 0.01m and ẋ0 = 0.

x0 = X cos φ
X cos φ = 0.01 (2.61)

To get ẋ0 lets expand Equation 2.60, then get the first time derivative and substitute t=0.
This leads to:

ẋ0 = −ζωn X cos φ + Xωd sin φ


0 = −(0.05)(20)X cos φ + X(19.975) sin φ
= −(0.05)(20)(0.01) + 19.975X sin φ
X sin φ = 0.000501 (2.62)
Vibrations: 2.2 Forced Vibrations 32

Equations 2.61 and 2.62 can be solved simultaneously to get:

X = −0.010012m
φ = 2.8680

Hence the total response becomes:


π
x = 0.010012e−0.05(20)t cos(19.975t − 2.868 × )
180
= 0.010012e−t cos(19.975t − 0.0501)

Response of an Under-Damped System Under Harmonic Motion of the Base

Sometimes, the base or support of a spring-mass-damper system undergoes harmonic motion as


shown in Figure 2.8.

Figure 2.8: Base Excitation

Let y denote the displacement of the base and x the displacement of the mass from the static
equilibrium.

The net elongation of the spring is x-y and the relative velocity between the two end s of the
damper is ẋ − ẏ.
Vibrations: 2.2 Forced Vibrations 33

The equation of motion can be shown to be:

mẍ + c(ẋ − ẏ) + k(x − y) = 0


mẍ + cẋ + kx = ky + cẏ
mẍ + cẋ + kx = kY sin ωt + cωY cos ωt (2.63)

Equation 2.63 shows that, giving the base an excitation of y = Y sin ωt is equivalent to applying
an harmonic force of magnitude kY sin ωt + cωY cos ωt to the system.

To get the particular solution, let:

kY sin ωt + cωY cos ωt = X1 cos(ωt − φ1 )


kY sin ωt + cωY cos ωt = X1 cos ωt cos φ1 + X1 sin ωt sin φ1 (2.64)

Equate the coefficients of sin ωt and also of cos ωt on both sides of Equation 2.64:

kY = X1 sin φ1 (2.65)
cωY = X1 cos φ1 (2.66)

Divide Equation 2.65 with Equation 2.66:


X1 sin φ1 kY
= = tan φ1
X1 cos φ1 cωY
hki
φ1 = tan−1 (2.67)

Squaring Equations 2.65 and 2.66 and adding them leads to:

X12 (sin2 φ1 + cos2 φ1 ) = Y 2 (k 2 + c2 ω 2 )



X 1 = Y k 2 + c2 ω 2 (2.68)

Therefore, the particular solution of Equation 2.63 is:


X1
xp = h i 21 cos(ωt − φ1 − φo )
(k 2 − mω 2 )2 + c2 ω 2

Y k 2 + c2 ω 2
= h i 21 cos(ωt − φ1 − φo ) (2.69)
(k 2 − mω 2 )2 + c2 ω 2

where φ was previously shown to be:


−1
h cω i
φo = tan
k 2 − mω 2
Vibrations: 2.2 Forced Vibrations 34

Equation 2.69 can be written in a more convenient form as:

xp = A cos(ωt − α) (2.70)

where:
s
A k 2 + c2 ω 2
=
Y (k 2 − mω 2 )2 + c2 ω 2

This ratio is called the displacement transmissibility (amplitude ratio) which gives the ratio of
the amplitudes of the steady state response (xp ) and the base motion (y).

Therefore the total response of a single under-damped degree of freedom vibrating system whose
base is harmonically excited by y = Y sin ωt is given as:

x = xh + xp

−ζωn t Y k 2 + c2 ω 2
x = Xe cos(ωd t − φ) + h i 21 cos(ωt − φ1 − φo ) (2.71)
2 2 2 2
(k − mω ) + c ω 2

where constants X and φ which are different from the ones for free-damped vibration can be
obtained by applying the initial conditions x0 and ẋ0 to the Equation 2.71.

Example 2.2.3. Figure 2.9 shows a model of a motor vehicle that can vibrate in the vertical
direction while traveling over a rough road. The vehicle has a mass of 1200kg. The suspension
system has a spring constant of 400kN/m and a damping ratio of ζ = 0.5. If the vehicle speed
is 20km/hr, determine the amplitude of the steady state motion of the vehicle. The road surface
varies sinusoidally with an amplitude of Y=0.05m and a wavelength of 6m.

Solution

The frequency (cycles per second) of the base motion can be shown to be:
v
f =
l
where v is the speed of the vehicle in m/s and l is the wave length (the length of one cycle of the
rough road).

Therefore, the frequency ω of the base excitation is:


2πv 2π × 20 × 1000
ω = 2πf = = = 5.81778rad/s
l 3600 × 6

The natural frequency ωn is:


r r
k 400 × 103
ωn = = = 18.2574rad/s
m 1200
Vibrations: 2.2 Forced Vibrations 35

Figure 2.9:

The damping constant:

c = 2mωn ζ = 2 × 1200 × 18.2574 × 0.05 = 21908.88N.s/m

The amplitude ratio is given as:


s
A k 2 + c2 ω 2
=
Y (k 2 − mω 2 )2 + c2 ω 2
v
u 4000002 + 21908.882 (5.81778)2
= uth i2 = 1.100965
2 2
400000 − 1200(5.81778) + 21908.88 (5.81778)2

Therefore the amplitude of the steady-state motion of the vehicle is:

A = 1.100965Y = 1.100965 × 0.05 = 0.055048m


Chapter 3

Vibrations of Multi-Degree of Freedom


Systems

3.1 Introduction

When a system requires more than one coordinate to describe its configuration, it is called a
multi-dof system or n-dof system where n is the number of coordinates required.

The n-dof system has n-natural frequencies, and for each of the natural frequencies, there corre-
sponds a natural state of vibration with a displacement configuration known as the normal mode.
Mathematical terms relating to these quantities are known as eigenvalues and eigenvectors. They
are established from n-simultaneous equations of motion of a system and posses certain dynamic
properties associated with the system.

Normal mode vibrations are free undamped vibrations that depend only on the mass and stiffness
of the system and how they are distributed. When vibrating at one of these normal modes,
all points in the system undergo simple harmonic motion that passes through their equilibrium
positions simultaneously.

To initiate a normal mode vibration, the system must be given specific initial conditions corre-
sponding to its normal mode.

3.2 Frequencies and Mode Shapes for Undamped Systems

Determination of the frequencies and mode shapes of a multi-dof vibrating system is very impor-
tant, since;

(a) it helps in tuning away the natural frequencies from the operating speed range of a system in
order to avoid resonance.
Vibrations: 3.2 Frequencies and Mode Shapes for Undamped Systems 37

(b) mode shapes show the relative displacements between the various elements of the system. The
mode shapes show the element that most affected by the given vibration mode. The natural
frequency of that mode can be altered by changing the stiffness or mass properties of that
element.

The first step in determining the natural frequencies and mode shapes of n-dof systems is to
determine the equations of motion and hence the mass M and stiffness K matrices of the system.
This can be achieved through the following three common methods

(a) From equations of motion using Newtons Second law of motion

(b) Using the influence coefficients

(c) From the equations of motion using Lagrangian equation.

3.2.1 Using Newton’s Second Law to Derive Equations of Motion

Consider a 3-dof mass-spring system shown in Figure 3.14 to illustrate the procedure that can be
adopted to derive the equations of motion of a multi-dof system using Newton’s second law of
motion.

Figure 3.1: Three dof mass-spring system.

STEP 1 Set up suitable coordinates to describe the positions of various point masses or rigid bodies
in the system.

Figure 3.2: Coordinates of a 3-dof mass-spring system


Vibrations: 3.2 Frequencies and Mode Shapes for Undamped Systems 38

STEP 2 Assuming x3 > x2 > x1 draw the free body diagram of each mass or rigid body in the system.

Figure 3.3: Free body diagrams of the masses

STEP 3 Apply Newton’s second law of motion to each mass or rigid body using D’Alembert’s prin-
ciple)

For mass m1 , m1 x¨1 + k1 x1 − k2 (x2 − x1 ) = 0


For mass m2 , m2 x¨2 + k2 (x2 − x1 ) − k3 (x3 − x2 ) = 0
For mass m3 , m3 x¨3 + k3 (x3 − x2 ) = 0

Simplifying,

m1 x¨1 + (k1 + k2 )x1 − k2 x2 = 0 (3.1)


m2 x¨2 − k2 x1 + (k2 + k3 )x2 − k3 x3 = 0 (3.2)
m3 x¨3 − k3 x2 + k3 x3 = 0 (3.3)

Equations 3.1 to 3.5 can be expressed in matrix form as:


       
m1 0 0  x¨1  k1 + k2 −k2 0  x1   0 
 0 m2 0  x¨2 +  −k2 k2 + k3 −k3  x2 = 0 (3.4)
0 0 m3 x¨3 0 −k3 k3 x3 0
     

or
M {x} + K{x} = {0}

We are interested in knowing whether all the masses can oscillate harmonically with the same
frequency but with different amplitudes. Each mass undergoes harmonic motion of the same
frequency, therefore, for each mode:

x1 = X1 sin ωt ⇒ ẍ1 = −ω 2 X1 sin ωt


x2 = X2 sin ωt ⇒ ẍ2 = −ω 2 X2 sin ωt
x3 = X3 sin ωt ⇒ ẍ3 = −ω 2 X3 sin ωt

where Xi = constants. Substituting these equations into the differential equations of motion, we
have:
− ω 2 M {X} + K{X} = {0} (3.5)
Vibrations: 3.2 Frequencies and Mode Shapes for Undamped Systems 39

or
[K − λM ]{X} = {0} (3.6)
where, M and K are both square symmetric matrices and λ is the eigenvalue related to natural
frequency by λ = f (ω 2 ).

Pre-multiplying both sides of Equation 3.6 by M−1 , we have another form of the equation.

[A − λI]{X} = {0} (3.7)

where A=M−1 K and is called the dynamic matrix.

The eigenvalues can be determined by setting the determinant of the characteristic matrix of
Equation 3.7, to zero. That is:
|A − λI| = 0 (3.8)

Example 3.2.1. For a simplified analysis of vibration of a mechanical system, a three degree-
of-freedom model, as shown in Figure 3.4, can be used.

(a) Derive the equations of motion of the system using the three-degree-of-freedom model.

(b) Using the equations of motion derived in part (a), find the natural frequencies and the mode
shapes of the system.

Figure 3.4:

Solution
Vibrations: 3.2 Frequencies and Mode Shapes for Undamped Systems 40

Figure 3.5:

(a) Assuming x3 > x2 > x1 the free body diagram of the system can be drawn as shown in Figure
3.5
Applying dynamic equilibrium on each mass:

mẍ1 + kx1 − k(x2 − x1 ) − 3k(x3 − x1 ) = 0


mẍ1 + 5kx1 − kx2 − 3kx3 = 0 (3.9)
2mẍ2 + 2kx2 + k(x2 − x1 ) − k(x3 − x2 ) = 0
2mẍ2 − kx1 + 4kx2 − kx3 = 0 (3.10)
mẍ3 + k(x3 − x2 ) + 3k(x3 − x1 ) + kx3 = 0
mẍ3 − 3kx1 − kx2 + 5kx3 = 0 (3.11)

Equations 3.9, 3.10 and 3.11 can be represented in matrix form to get equation of motion of
the system:
       
m 0 0  x¨1  5k −k −3k  x1   0 
 0 2m 0  x¨2 +  −k 4k −k  x2 = 0 (3.12)
0 0 m x¨3 −3k −k 5k x3 0
     
Vibrations: 3.2 Frequencies and Mode Shapes for Undamped Systems 41

(b) From the equation of motion derive in (a) above, the mass and stiffness matrices are:
   
m 0 0 1 0 0
M =  0 2m 0  = m  0 2 0 
0 0 m 0 0 1
   
5k −k −3k 5 −1 −3
K =  −k 4k −k  = k  −1 4 −1 
−3k −k 5k −3 −1 5
 
1 0 0
−1 1 
M = 0 0.5 0 
m
0 0 1
    
1 0 0 5 −1 −3 5 −1 −3
k  k 
A = M −1 K = 0 0.5 0   −1 4 −1  = −0.5 2 −0.5 
m m
0 0 1 −3 −1 5 −3 −1 5

The characteristic matrix equation becomes:

[A − ω 2 I]{X} = {0}
        
5 −1 −3 1 0 0  X1  0
 k  −0.5 2 −0.5  − ω 2  0 1 0   X2 = 0
m
−3 −1 5 0 0 1 X3 0
   
       
5 −1 −3 2 1 0 0  X1  0
 −0.5 2 −0.5  − ω m  0 1 0   X2 = 0
k
−3 −1 5 0 0 1 X3 0
   
(3.13)
ω2 m
Let λ = k
, and set the characteristic determinant to zero.
   
5 −1 −3 1 0 0
 −0.5 2 −0.5  − λ  0 1 0  = 0
−3 −1 5 0 0 1
5 − λ −1 −3
−0.5 2 − λ −0.5 = 0
−3 −1 5 − λ
     
(5 − λ) (2 − λ)(5 − λ) − 0.5 + 1 − 0.5(5 − λ) − 1.5 − 3 0.5 + 3(2 − λ) = 0
λ3 − 12λ2 + 35λ − 24 = 0
(λ − 1)(λ − 3)(λ − 8) = 0

Solving for the eigenvalues,

λ − 1 = 0 ⇒ λ1 = 1
λ − 3 = 0 ⇒ λ2 = 3
λ − 8 = 0 ⇒ λ3 = 8
Vibrations: 3.2 Frequencies and Mode Shapes for Undamped Systems 42

This leads to three natural frequencies as:


q
k
λ1 = 1 ⇒ ω1 = m
q
k
λ2 = 3 ⇒ ω2 = 1.732 m
q
k
λ3 = 8 ⇒ ω3 = 2.828 m

Once the natural frequencies are known, the mode shapes or eigenvectors can be calculated.
The mode shapes represent the relative displacement between the elements or the normalized
amplitude ratios of all the masses corresponding to each natural frequency.
The eigenvector corresponding to the eigenvalue λi can be determined from co-factors of any
row of the characteristic matrix [A − λi I] where i is the mode shape.

1st mode

The characteristic matrix is given by [A − λ1 I]:


 
4 −1 −3
=  −0.5 1 −0.5 
−3 −1 4

1 −0.5
c11 = (−1)2 = 3.5
−1 4
−0.5 −0.5
c12 = (−1)3 = 3.5
−3 4
−0.5 1
c13 = (−1)4 = 3.5
−3 −1
The first mode is therefore given by:
     
 c11   3.5   1.000 
α c12 = 3.5 → 1.000 1
c13 3.5 1.000
     

and the corresponding mode shape is show in Figure ??

2nd mode

The characteristic matrix is given by [A − λ2 I]:


 
2 −1 −3
=  −0.5 −1 −0.5 
−3 −1 2

−1 −0.5
c11 = (−1)2 = −2.5
−1 2
1
In this case, normalization of the modes is done by dividing the elements with the first element
Vibrations: 3.2 Frequencies and Mode Shapes for Undamped Systems 43

−0.5 −0.5
c12 = (−1)3 = 2.5
−3 2
−0.5 −1
c13 = (−1)4 = −2.5
−3 −1
The second mode is therefore given by:
     
 c11   −2.5   1.000 
α c12 = 2.5 → −1.000
c13 −2.5 1.000
     

and the corresponding mode shape is show in Figure ??

3rd mode

The characteristic matrix is given by [A − λ3 I]:


 
−3 −1 −3
=  −0.5 −6 −0.5 
−3 −1 −3

−6 −0.5
c11 = (−1)2 = 17.5
−1 −3
−0.5 −0.5
c12 = (−1)3 =0
−3 −3
−0.5 −6
c13 = (−1)4 = −17.5 ∴ c13 = 1
−3 −1
The third mode is therefore given by:
     
 c11   17.5   1 
α c12 = 0 → 0
c13 −17.5 −1
     

3.2.2 Influence Coefficients

The equations of motion of multi-DOF systems can be written in terms of influence coefficients.
The influence coefficients associated with the stiffness and mass matrices are know as the stiffness
and inertia influence coefficients.

In some cases, it is more convenient to rewrite the equations of motion using the inverse of
the stiffness matrix (known as the flexibility matrix) or the inverse of the mass matrix. The
influence coefficients corresponding to the flexibility matrix are known as the flexibility influence
coefficients.
Vibrations: 3.2 Frequencies and Mode Shapes for Undamped Systems 44

Stiffness Influence Coefficients

The stiffness influence coefficient denoted by kij is defined as the force at point i due to a unit
displacement at j with all other displacements = 0.2

Example 3.2.2. Determine the stiffness influence coefficients for the system shown in Figure 3.6

Solution

The stiffness influence coefficients kij can be determined from the spring stiffnesses k1 , k2 and k3
as follows:

• Draw the free body diagram of the system.

• Apply unit displacement at 1, x1 = 1, with x2 = x3 = 0 and apply conditions of static


equilibrium to each body.

Figure 3.6: Free body diagram

X
F = 0 ⇒ F1 − k1 + k2 (0 − 1) = 0
∴ F1 = k1 + k2
F2 − k2 (0 − 1) + k3 (0 − 0) = 0 ⇒ F2 = −k2
F3 − k3 (0 − 0) = 0 ⇒ F3 = 0

The forces F1 , F2 , F3 are found from the first column of the stiffness matrix.
     
F1 k11 k1 + k2 0 0
 F2  =  k21  =  −k2 0 0 
F3 k31 0 0 0
Again, apply unit displacement at station 2, x2 = 1, with x1 = x3 = 0.
X
F = 0 ⇒ F1 − k1 (0) + k2 (1 − 0) = 0
∴ F1 = −k2
F2 − k2 (1 − 0) + k3 (0 − 1) = 0 ⇒ F2 = k2 + k3
F3 − k3 (0 − 1) = 0 ⇒ F3 = −k3
2
For a simple linear spring, the force necessary to cause unit elongation is known as the stiffness of the spring.
Vibrations: 3.2 Frequencies and Mode Shapes for Undamped Systems 45

The forces F1 , F2 , F3 are found from the second column of the stiffness matrix.
     
F1 k12 0 −k2 0
 F2  =  k22  =  0 k2 + k3 0 
F3 k32 0 −k3 0
Again, apply unit displacement at station 3, x2 = 1, with x1 = x3 = 0.

X
F = 0 ⇒ F1 − k1 (0) + k2 (0 − 0) = 0
∴ F1 = 0
F2 − k2 (0 − 0) + k3 (1 − 0) = 0 ⇒ F2 = −k3
F3 − k3 (1 − 0) = 0 ⇒ F3 = k3

The forces F1 , F2 , F3 are found from the third column of the stiffness matrix.
     
F1 k13 0 0 0
 F2  =  k23  =  0 0 −k3 
F3 k33 0 0 k3
The three matrices are then assembled to form the complete stiffness matrix.
 
k1 + k2 −k2 0
K =  −k2 k2 + k3 −k3 
0 −k3 k3

Flexibility Influence Coefficients

Flexibility is the inverse of stiffness. Displacement can be written in terms of the force and stiffness
by equation 3.14

F
x= (3.14)
k
In matrix form,

X = K −1 F
= AF

where,  
a11 a12 a13
A =  a21 a22 a23 
a31 a32 a33
The flexibility influence coefficient aij is defined as the displacement at i due to a unit force applied
at j with other forces =0.

Example 3.2.3. A three-storey building can be modeled as shown in Figure 3.7. Determine the
flexibility matrix of the system.
Vibrations: 3.2 Frequencies and Mode Shapes for Undamped Systems 46

Figure 3.7: Three-storey building

Solution

From the free body diagram, first apply a unit force at station 1, F1 = 1, with F2 = F3 = 0 and
apply static equilibrium conditions.

Figure 3.8: Free body diagram

0 = k3 (x3 − x2 ) ⇒ x3 = x2
0 = k2 (x2 − x1 ) − k3 (x3 − x2 ) ⇒ x2 = x1
1
1 − k1 x1 + k2 (x2 − x1) = 0 ⇒ x1 =
k1
x1 = x2 = x3

The displacements x1 , x2 , x3 are found from the third column of the stiffness matrix.
     1 
x1 a11 k1
0 0
 x2  =  a21  =  1 0 0 
k1
1
x3 a31 k1
0 0
Vibrations: 3.2 Frequencies and Mode Shapes for Undamped Systems 47

Repeat the procedure for the other stations and obtain the complete flexibility matrix
 1 1 1 
k1 k1 k1
1 1 1 1 1
K= k1 k1
+ k2 k1
+ k2

1 1 1 1 1 1
k1 k1
+ k2 k1
+ k2
+ k3

Tutorial 2
Q1 Using Newton’s Second Law of motion, derive the equation of motion in matrix form for the
system shown in Figure 3.9

Figure 3.9:

Q2 Consider the two-degree-of-freedom system shown in Figure 3.10 with m1 = m2 = 1kg and
k1 = k2 = 4N/m The masses m1 and m2 move on a rough surface for which the equivalent
viscous damping constants can be assumed as c1 = c2 = 2Ns/m.

Figure 3.10:

(a) Derive the equations of motion of the system and represent them in matrix form.
(b) Find the natural frequencies and mode shapes of the undamped system.

Q3 For a simplified analysis of the vibration of an airplane in the vertical direction, a three
degree- of-freedom model, as shown in Figure 3.11, can be used. The three masses indicate the
masses of the two wings (m1 = m3 = m) and the fuselage (m2 = 5m). The stiffness k1 = k2 = k
correspond to the bending stiffnesses of the two wings, which can be modeled as cantilever
beams so that k1 = k2 = k = 3EIl3
.

(a) Derive the equations of motion of the airplane using the three-degree-of-freedom model.
Vibrations: 3.2 Frequencies and Mode Shapes for Undamped Systems 48

(b) Using the equations of motion derived in part (a), find the natural frequencies and mode
shapes of the airplane. Give an interpretation of the results.

Figure 3.11:

Q4 Find the flexibility and stiffness influence coefficients of the system shown in Figure 3.12.
Also, derive the equations of motion of the system.

Figure 3.12:
Vibrations: 3.2 Frequencies and Mode Shapes for Undamped Systems 49

Q5 A simplified model of the main landing gear system of a small airplane is shown in Figure
3.13 with m1 = 100kg, m2 = 5000kg, k1 = 104 N/m and k2 = 106 N/m.

(a) Derive the equations of motion of the system.


(b) Find the natural frequencies and the mode shapes of the system.

Figure 3.13:
50
Vibrations: 3.3 Numerical Methods of Finding the Natural Frequencies and Mode Shapes

3.3 Numerical Methods of Finding the Natural Frequen-


cies and Mode Shapes

In the previous section, the natural frequencies (eigenvalues) and natural mode shapes (eigenvec-
tors) of a multi-dof system were found analytically by setting the characteristic determinant to
zero.

Although this method is exact, the expansion of the characteristic determinant and the solution
of the resulting n-th degree polynomial equation to obtain eigenvalues and eigenvectors is very
tedious especially for large values of n (degree of freedom).

Several numerical methods have been developed for computing the natural frequencies and mode
shapes for a multi-dof vibrating systems. These methods can be implemented as computer algo-
rithms to analyze even a system with very large degree of freedom. These methods include:

(a) Matrix iteration method

(b) Holzer’s method

(c) Dunkerleys method

(d) Rayleigh’s method

(e) Jacobi’s method

In this section we will consider the Matrix iteration method and the Holzer’s method.

3.3.1 Matrix Iteration

The eigenvalues and eigenvectors of a multi-dof system can be determined by matrix iteration
procedure. The method is applicable to the equations of motion formulated by either the flexibility
or the stiffness matrices. The equation of motion of an undamped vibrating multi-dof system has
been shown to be represented as:

M {x} + K{x} = {0}


This equation can also be represented as:

− ω 2 M {X} + K{X} = {0}

which can be rewritten as:

[A − λI]{X} = {0}
A{X} = λI{X}
A{X} = λ{X} (3.15)
51
Vibrations: 3.3 Numerical Methods of Finding the Natural Frequencies and Mode Shapes

where λ = ω 2 and A=M−1 K called the dynamic matrix.

The iteration is started by assuming a trial vector for the vector X on the left hand side of
equation 3.15 which results in a column vector. This vector is then normalized. The procedure
is then repeated with the normalized vector until the elements stabilize to a definite pattern
(converges). In this case, since the dynamic matrix A uses stiffness matrix K, the solution will
converge to the highest mode of vibration as it will be seen in the following example.

Example 3.3.1. Consider the three-dof mass-spring system shown in Figure 3.14

Figure 3.14: Three dof mass-spring system.

If m1 =m, m2 =m, m3 =m and k1 =k, k2 =k, k3 =2k. Determine the highest natural frequency and
the mode shape of the system using matrix iteration.

Solution

The equation of motion of the system can be shown to be:


       
m1 0 0  x¨1  k1 + k2 −k2 0  x1   0 
 0 m2 0  x¨2 +  −k2 k2 + k3 −k3  x2 = 0 (3.16)
0 0 m3 x¨3 0 −k3 k3 x3 0
     

or
M {x} + K{x} = {0}

where:
   
m1 0 0 1 0 0
M =  0 m2 0  = m  0 1 0  (3.17)
0 0 m3 0 0 1
     
k1 + k2 −k2 0 k + k −k 0 2 −1 0
K =  −k2 k2 + k3 −k3  =  −k k + 2k −2k  = k  −1 3 −2  (3.18)
0 −k3 k3 0 −2k 2k 0 −2 2
 
1 0 0
1 
M −1 = 0 1 0 (3.19)
m
0 0 1
    
1 0 0 2 −1 0 2 −1 0
k  k 
A = M −1 K = 0 1 0   −1 3 −2  = −1 3 −2  (3.20)
m m
0 0 1 0 −2 2 0 −2 2
52
Vibrations: 3.3 Numerical Methods of Finding the Natural Frequencies and Mode Shapes

   AX  = ω2X
 
2 −1 0 X1 X1
k 
−1 3 −2   X2  = ω 2  X2 
m
0 −2 2 X3 X3
    
2 −1 0 X1 2 X1
 −1 ω m
3 −2   X2  = X2 
k
0 −2 2 X3 X3
    
2 −1 0 X1 X1
 −1 3 −2   X2  = λ  X2 
0 −2 2 X3 X3
mω 2
Where λ = k

Starting with an arbitrary trial vector, (1 1 1)T , begin iteration:


      
2 −1 0 1 1 1
AX1 = −1 3 −2
   1 =  0 =1 0
 
0 −2 2 1 0 0
      
2 −1 0 1 2 1
AX2 = −1 3 −2
   0 =  −1  = 2  −0.5 
0 −2 2 0 0 0
      
2 −1 0 1 2.5 1
AX3 =  −1 3 −2   −0.5  =  −2.5  = 2.5  −1 
0 −2 2 0 1 0.4
      
2 −1 0 1 3 1
AX4 =  −1 3 −2   −1  =  −4.8  = 3  −1.6 
0 −2 2 0.4 2.8 0.933
      
2 −1 0 1 3.6 1
AX5 =  −1 3 −2   −.6  =  −7.666  = 3.6  −2.13 
0 −2 2 0.933 5.067 1.407
      
2 −1 0 1 4.13 1
AX6 =  −1 3 −2   −2.13  =  −10.204  = 4.13  −2.471 
0 −2 2 1.407 7.074 1.713
      
2 −1 0 1 4.471 1
AX7 =  −1 3 −2   −2.471  =  −11.838  = 4.471  −2.648 
0 −2 2 1.713 8.368 1.872
      
2 −1 0 1 4.648 1
AX8 =  −1 3 −2   −2.648  =  −12.687  = 4.648  −2.73 
0 −2 2 1.872 9.039 1.945
      
2 −1 0 1 4.73 1
AX9 =  −1 3 −2   −2.73  =  −13.078  = 4.73  −2.765 
0 −2 2 1.945 9.349 1.977
      
2 −1 0 1 4.765 1
AX10 =  −1 3 −2   −2.765  =  −13.249  = 4.765  −2.78 
0 −2 2 1.977 9.484 1.99
53
Vibrations: 3.3 Numerical Methods of Finding the Natural Frequencies and Mode Shapes

      
2 −1 0 1 4.78 1
AX11 = −1
 3 −2   −2.78  =  −13.322  = 4.78  −2.73 
0 −2 2 1.99 9.541 1.945
      
2 −1 0 1 4.79 1
AX12 =  −1 3 −2   −2.73  =  −13.352  = 4.79  −2.79 
0 −2 2 1.945 9.566 2.00
     
1 X1 2 1
mω 
∴ 4.79  −2.79  = λ  X2  = −2.79 
k
2.00 X3 2.00
r
mω32 k
⇒ = 4.79 ∴ ω3 = 2.189
k m

Convergence to Lower Modes of Vibration

As seen in the above example, when characteristic matrix is formulated using the stiffness matrix
K, the matrix iteration procedure always converges to the largest natural frequency. However in
vibration analysis, the lower modes of vibration are generally of greater interest than the higher
modes.

If the flexibility matrix (a = K −1 ) is used instead of the stiffness matrix, then the iteration
procedure converges to the lowest mode of vibration. If the lowest mode is absent in the assumed
deflection, the iteration will then converge to the next lowest mode (second mode)

In terms of the flexibility matrix [a]=[k]−1 , the equation of normal mode vibration is:

ĀX = λ̄X (3.21)


where Ā = [a][m] = K −1 M
1
λ̄ = 2
ω
Example 3.3.2. For the problem in example 3.3.1, obtain the lowest natural frequency and the
corresponding mode shape vector by matrix iteration.

Solution

The mass and flexibility matrices for the system are:


   1 1 1   
1 0 0 k1 k1 k1 1 1 1
M =m 0 1 0  [a] = K −1  k11 k11 + k12 1
+ 1 = 11 2 2
k1 k2
k
0 0 1 1 1
k1 k1
+ k12 1
k1
+ 1
k2
+ 1
k3
1 2 52

The dynamic matrix is given by:


    
1 1 1 1 0 0 1 1 1
m m
Ā = [a][m] = 1 2 2  0 1 0  = 1 2 2 
k 5 k
1 2 2 0 0 1 1 2 2.5
54
Vibrations: 3.3 Numerical Methods of Finding the Natural Frequencies and Mode Shapes

   ĀX
 = λ̄X 
1 1 1 X1 X1
m 1
1 2 2   X2  = 2  X2 
k ω
1 2 2.5 X3 X3
    
1 1 1 X1 X1
1 k 
2 2   X2  = X2 
mω 2
1 2 2.5 X3 X3
    
1 1 1 X1 X1
1 2 2   X2  = λ̄  X2 
1 2 2.5 X3 X3
k
Where λ̄ = mω 2
.

Starting with a trial vector, (1 1 1)T , begin iteration:


      
1 1 1 1 3.0 1.0
ĀX1 =  1 2 2   1  =  5.0  = 3  1.667 
1 2 2.5 1 5.5 1.833
      
1 1 1 1.0 4.5 1.0
ĀX2 =  1 2 2   1.667  =  8  = 4.5  1.778 
1 2 2.5 1.833 8.917 1.981
      
1 1 1 1.0 4.759 1.0
ĀX3 =  1 2 2   1.778  =  8.518  = 4.759  1.790 
1 2 2.5 1.981 9.509 1.998
      
1 1 1 1.0 4.788 1.0
ĀX4 =  1 2 2   1.790  =  8.576  = 4.788  1.7911 
1 2 2.5 1.998 9.575 2.0
      
1 1 1 1.0 4.791 1.0
ĀX5 =  1 2 2   1.7911  =  8.582  = 4.791  1.791 
1 2 2.5 2.0 9.582 2.0
      
1 1 1 1.0 4.791 1.0
ĀX6 =  1 2 2   1.791  =  8.582  = 4.791  1.791 
1 2 2.5 2.0 9.582 2.0
     
1.0 X1 1.0
k 
∴ 4.791  1.791  = λ̄  X2  = 1.791 
mω 2
2.0 X3 2.0
r
k k
⇒ 2
= 4.791 ∴ ω1 = 0.457
mω1 m

3.3.2 Holzer’s Method

Holzer’s method is a trial-and-error scheme used to find the natural frequencies of undamped and
damped semi-definite (free-free or unrestrained) vibrating systems involving linear and angular
displacements. The method can also be programmed for computer applications.
55
Vibrations: 3.3 Numerical Methods of Finding the Natural Frequencies and Mode Shapes

This method is easily applied to determine the natural vibrations and vibration modes of a tor-
sional vibrating system.

Torsional Vibrations

Torsional vibrations are encountered in many machines, especially in power transmission systems
using rotating shafts or couplings where it can cause failures if not controlled.

Structures subjected to twisting moments also exhibit torsional vibrations. The torques generated
may not be smooth (e.g., internal combustion engines) or the component being driven may not
react to the torque smoothly (e.g., reciprocating compressors). Also, the components transmitting
the torque can generate non-smooth or alternating torques (e.g., worn gears, misaligned shafts).
Because the components in power transmission systems are not infinitely stiff these alternating
torques cause vibration about the axis of rotation.

If a rigid body oscillates about a specific reference axis, the resulting motion is called torsional
vibration. In this case, the displacement of the body is measured in terms of an angular coordinate
θ.

In a torsional vibration problem, the restoring moment may be due to the torsion of an elastic
member or due to the unbalanced moment of a force or couple.

Figure 3.15 shows a disc, which has a mass moment of inertia Jo mounted at one end of a solid
circular shaft, the other end of which is fixed.

Figure 3.15: Torsional vibration of a disc.


56
Vibrations: 3.3 Numerical Methods of Finding the Natural Frequencies and Mode Shapes

Let the angular rotation of the disc about the axis of the shaft be θ which also represents the
shaft’s angle of twist. From the torsional formula:
T Gθ
=
Io l
the torque that produces an angle of twist θ is given as:
GIo
T = θ (3.22)
l
where G is the modulus of rigidity of the shaft material, l the length of the shaft and Io is the
polar moment of area of the shaft given as:
πd4
Io =
32
If the disc is displaced by θrads from its equilibrium position, the shaft will therefore provide
a restoring torque T given in Equation 3.22. Thus the shaft acts as a torsional spring with a
torsional spring constant given as:
T GIo Gπd4
kt = = =
θ l 32l

By applying dynamic equilibrium on the disc after displacement, we get an ode given as:

Jo θ̈ + kt θ = 0
kt
θ̈ + θ = 0 (3.23)
Jo
kt
where Jo
= ωn2 , the square of the natural frequency of the torsional vibration of the disc.

Holzer’s Method

When an undamped system is vibrating freely at any one of its natural frequencies, no external
force, torque or moment is necessary to maintain the vibration.

Holzer proposed a method of calculating the natural frequencies and modal vectors of torsional
systems by assuming a natural frequency and starting with a unit amplitude at one end of the
system and progressively calculating the torque and angular displacement to the other end.

The frequencies that result in zero external torque or compatible boundary conditions at the other
end are the natural frequencies of the system. The method can be applied to any lumped-mass
system, linear mass-spring systems, beams modeled by discrete masses and beams springs.

Consider a free-free system shown in Figure 3.16. The equations of motion are given by:

J1 θ¨1 + k1 (θ1 − θ2 ) = 0
J2 θ¨2 + k1 (θ2 − θ1 ) − k2 (θ3 − θ2 ) = 0
J3 θ¨3 + k2 (θ3 − θ2 ) = 0
57
Vibrations: 3.3 Numerical Methods of Finding the Natural Frequencies and Mode Shapes

Figure 3.16: Free-free system

Assuming simple harmonic motion,


     ¨   
θ1 Θ1 θ1 Θ1
 θ2  =  Θ2  sin ωt ⇒  θ¨2  = −ω 2  Θ2  sin ωt
θ3 Θ3 θ¨3 Θ3

Substituting these equations in the equations of motion,

ω 2 J1 Θ1 = k1 (Θ1 − Θ2 ) (3.24)
ω 2 J2 Θ2 = k1 (Θ2 − Θ1 ) − k2 (Θ3 − Θ2 ) (3.25)
ω 2 J3 Θ3 = k2 (Θ3 − Θ2 ) (3.26)

Adding equations 3.24 - 3.26 gives:

ω 2 J1 Θ1 + ω 2 J2 Θ2 + ω 2 J3 Θ3 = k1 (Θ1 − Θ2 ) + k1 (Θ2 − Θ1 ) − k2 (Θ3 − Θ2 ) + k2 (Θ3 − Θ2 ) = 0


Xn
ωi2 Ji Θi = 0 (3.27)
i=1

Equation 3.27 states that the sum of the inertia torques of a free-free (semi-definite) system must
be zero.

In Holzers method, the amplitude of the first element is assumed to be unity Θ1 = 1 and the
equation of the total inertia torque on the system determined as a function of ω

Θ1 = 1
ω 2 J1 Θ1
Θ2 = Θ1 − =
k1
ω2  
Θ3 = Θ2 − J1 Θ1 + J2 Θ2
k2

It is seen that for n-disc system,


i−1
ω2 X
Θi = Θi−1 − Jj Θj
ki−1 j=1
58
Vibrations: 3.3 Numerical Methods of Finding the Natural Frequencies and Mode Shapes

Therefore using Equation 3.27, we have:

2 2
 ω 2 J1 Θ1  2
h ω2  i
0 = ω J1 Θ1 + ω J2 Θ1 − + ω J3 Θ2 − J1 Θ1 + J2 Θ2
k1 k2

Noting that Θ = 1 and expanding Equation 3.28 we get:


J1 J2 J3 6 h J1 J2 J1 J3 J1 J3 J2 J3 i 4
Text = ω − + + + ω + (J1 + J2 + J3 )ω 2 = 0 (3.28)
k1 k2 k1 k1 k2 k2

A graph of ω versus Text is plotted and the values of ω when Text = 0 are the natural frequencies
of the system. The angular displacements Θi corresponding to the natural frequencies are the
natural modes.

The first natural frequency is zero and corresponds to the rigid mode vibration when the system
rotates as a rigid body.

Example 3.3.3. For the system shown in Figure 3.16, k1 = 0.1 × 106 Nm/rad, 0.2 × 106 Nm/rad,
J1 =5.0 kgm2 , J1 =11.0 kgm2 and J3 =22.0 kgm2 . Determine the natural frequencies and natural
modes for the system, given that the highest natural frequency is less than 210 rad/s.

ω1 = 0
ω2 = 123.66rad/s
ω3 = 202.66rad/s

The natural modes are given by:


 
1
X1 =  1 (3.29)
1
 
1.000
X2 =  0.235  (3.30)
−0.345
 
1.000
X3 =  −1.054  (3.31)
0.299
59
Vibrations: 3.3 Numerical Methods of Finding the Natural Frequencies and Mode Shapes

C:\MATLAB6p5\work\holz_table.txt
February 9, Holzers
Table 3.1: 2009 table
omega (rad/s) Torque (Nm)
============= ===========
0.00 10.000000
20.00 30.000000
40.00 50.000000
60.00 70.000000
80.00 90.000000
100.00 110.000000
120.00 130.000000
140.00 150.000000
160.00 170.000000
180.00 190.000000
200.00 210.000000
220.00 0.000000
3765.96 14658.272000
31482.00 52318.208000
74632.81 95429.088000
111443.66 119386.112000
116222.08 99500.000000
67721.34 20754.432000
-39708.16 -109648.672000
-182179.69 -246956.032000
-289543.18 -290742.048000
-225870.30 -64000.000000
232848.22 710554.592000

5
x 10
8

4
Text Nm/rad

-2

-4
0 50 100 150 200 250
ω (rad/s)

Figure 3.17: Holzers graph

Example 3.3.4. The schematic diagram of a marine engine connected to a propeller through gears
if shown in Figure 3.18. The mass moment of inertias of the flywheel, engine, gear1, gear2 and
60
Vibrations: 3.3 Numerical Methods of Finding the Natural Frequencies and Mode Shapes

the propeller are respectively Jf = 9000kg.m2 , Je = 1000kg.m2 , Jg1 = 250kg.m2 Jg2 = 150kg.m2
and Jp = 2000kg.m2 . The shear modulus of rigidity of steel is 80×109 Pa

(a) Find by equating the determinant of the dynamic matrix to zero, the natural frequencies and
mode shapes if the flywheel is considered fixed (stationary) since its mass moment of inertia is
very large compared to that of other rotors. Check your answers by using the matrix iteration
method.

(b) Find using the Holzer’s method the natural frequencies and the mode shapes by now considering
the flywheel as a rotating rotor. Check your answers by using the matrix iteration method.

Figure 3.18:

Solution

(a) Since the distance between the engine and the gears is small, then the three components can
be replaced by a single rotor. This will lead to a 2-dof system in torsion.
Lets reference the mass moment of inertia of gear2 and the flywheel with respect to the engine
be equivalent mass moment of inertia. Since gears 1 and 2 have 40 and 20 teeth, shaft 2
rotates at twice the speed of shaft 1. Thus the mass moments of inertia of gear 2 and the
propeller, referred to the engine, are given by:

Jg2 |Eq = n2 Jg2 = 22 × 150 = 600kg.m2


Jp |Eq = n2 Jp = 22 × 2000 = 8000kg.m2

Now the inertias of the engine, gear1 and gear2 are arranged in parallel, hence the equivalent
mass moment of inertia becomes:

J1 = Je + Jg1 + Jg2 |Eq = 1000 + 250 + 600 = 1850kg.m2


61
Vibrations: 3.3 Numerical Methods of Finding the Natural Frequencies and Mode Shapes

Since the length of shaft 2 is not negligible, the propeller is considered to be a rotor connected
at the end of shaft 2 with J2 = Jp |Eq = 8000kg.m2 as shown in Figure 19(a). Assuming
θ2 > θ1 , the the free body diagram of the system is as shown in Figure 19(b)

(a)

(b)

Figure 3.19:

The torsional stiffnesses are:


GIo Gπd41 80 × 109 × π × 0.14
k = = = = = 981, 750.0N.m/rad
l 32L1 32 × 0.8
Gπd42 80 × 109 × π × 0.154
k = = = = 3, 976, 078.5N.m/rad
32L2 32 × 1.0

From the free body diagram, the equations of motion are obtained as:

J1 θ¨1 + (k1 + k2 )θ1 − k2 θ2 = 0


J2 θ¨1 − k2 θ1 + k2 θ2 = 0

which can be written in matrix form as:

θ¨1
       
J1 0 k1 + k2 −k2 θ1 0
¨ + = (3.32)
0 J2 θ2 −k2 k2 θ2 0
or
J{θ} + K{θ} = {0}
62
Vibrations: 3.3 Numerical Methods of Finding the Natural Frequencies and Mode Shapes

At natural frequencies each rotor undergoes harmonic motion of the same frequency, therefore:

− ω 2 J{θ} + K{θ} = {0} (3.33)

or
[K − λJ]{θ} = {0} (3.34)
where λ = ω 2 . Pre-multiplying both sides of this equation by J−1 , we have another form of
the equation.
[A − λI]{θ} = {0} (3.35)
where A=J−1 K and is the dynamic matrix.
Setting the determinant of the characteristic matrix of this equation to zero, we can get the
eigenvalues. That is:
|A − λI| = 0 (3.36)

 
1850 0
J =
0 8000
 
4.96 × 106 −3.98 × 106
K =
−3.98 × 106 3.98 × 106
 1 
−1 1850
0
J = 1
0 8000
 1    
−1 1850
0 4.96 × 106 −3.98 × 106 2681.1 −2151.35
A = J K= 1 =
0 8000 −3.98 × 106 3.98 × 106 −497.5 497.5

The characteristic matrix equation becomes:


   
2681.1 −2151.35 1 0
−λ = 0
−497.5 497.5 0 1
2681.1 − λ −2151.35
= 0
−497.5 497.5 − λ

Solving for the eigenvalues, leads to two natural frequencies as:



λ1 = 85.312 ⇒ ω1 =√ 85.312 = 9.236rad/s
λ2 = 3091.6 ⇒ ω2 = 3091.6 = 55.6022rad/s
The corresponding eigenvectors which give the mode shapes are:
 
1
{θ1 } =
1.2072
 
1
{θ2 } =
−0.1916
(3.37)
63
Vibrations: 3.3 Numerical Methods of Finding the Natural Frequencies and Mode Shapes

Tutorial 3
Q1 The arrangement of the compressor, turbine, and generator in a thermal power plant is shown
in Figure 3.20. Find the natural frequencies and mode shapes of the system using the following
methods: [ω1 =0rad/s, ω2 =707.5rad/s and ω3 =1224.7rad/s]

(a) Matrix iteration method


(b) Holzer’s method

Figure 3.20:

Q2 Three freight cars are coupled by two springs, as shown in Figure 3.21. Using Holzer’s method,
find the natural frequencies
q and modeq shapes of the system if m1 = m2 = m3 = m and
k1 = k2 = k [ω1 = 0, ω2 = m and 3k
k
m
]

Figure 3.21:

Q3 A uniform shaft carries three rotors as shown in Figure 3.22 with mass moments of inertia
of J1 = 5kg.m2 , J2 = 15kg.m2 and J3 = 25kg.m2 . The torsional stiffnesses of the segments
between the rotors are given by kt1 =20000N.m/rad and kt2 =60000N.m/rad Determine the
natural frequencies and mode shapes of the system using Holzer’s method.

Figure 3.22:
64
Vibrations: 3.3 Numerical Methods of Finding the Natural Frequencies and Mode Shapes

Q4 A turbine is connected to an electric generator through gears, as shown in Figure 3.23. The
mass moments of inertia of the turbine, generator, gear 1, and gear 2 are given, respectively
3000kg.m2 , 2000kg.m2 , 500 kg.m2 , and 1000kg.m2 . Shafts 1 and 2 are made of steel and have
diameters 30cm and 10cm and lengths 2.0m and 1.0m, respectively. The shear modulus of
rigidity of steel is 80×109 Pa. Find the natural frequencies of the system and the mode shapes.

Figure 3.23:
Chapter 4

Transient Vibrations

4.1 Introduction

Transient vibration is defined as a temporarily sustained vibration of a mechanical system. The


steady state vibrations are generally not produced. Such oscillations take place at the natural
frequency of the system with the amplitude varying depending on the type of excitation.

Excitations on a vibrating system can be periodic or non-periodic. An example of periodic excita-


tion is the harmonic excitation which was considered in Chapter 2. The non-periodic excitations
include; a suddenly applied constant excitation (called a step-excitation), a linearly increasing
excitation (called a ramp-excitation), and an exponentially varying force.

A non-periodic forcing function may be acting for a short, long, or infinite duration. A forcing
function or excitation of short duration compared to the natural time period of the system is
called a shock (transient loading).

Transient loading, also known as impact, or mechanical shock, is a non-periodic excitation, which
is characterized by a sudden and severe application.

In real life, mechanical shock is very common. Some examples of shock could be a forging hammer,
an automobile passing trough a road bump/pot hole, the free drop of an item from a height, ground
vibration of a building frame during an earthquake, the motion imparted by a cam to the follower,
among others.

In the analysis of systems involving mechanical shock, the forcing function (displacement, velocity,
acceleration or force) of such system is usually idealized as a step or pulse function.

The response of a system subjected to any type of non-periodic force is commonly found using
the following methods:

(a) Representing the excitation by a Fourier integral.

(b) Convolution integral.


Vibrations: 4.2 Impulse Excitations 66

(c) Laplace transform.

(d) Numerical methods.

4.2 Impulse Excitations

A non-periodic exciting force usually has a magnitude that varies with time; it acts for a specified
period and then stops.

The simplest form of a non-periodic exciting force is the impulsive force. An impulsive force has
a large magnitude F which acts for a very short time ∆t.

Recall the principle of impulse and momentum in dynamics which states that:

F ∆t = F̂ = mẋ2 − mẋ1 (4.1)

Since F is large and ∆t is too small, then a unit impulse F̂ = f = 1 can be assumed to act at
t=0, such that;
Z t+∆t
f = lim F dt = 1
∆t →0 t

Figure 4.1: Unit impulse at t = 0


If the unit impulse is applied at t = τ , the unit impulse can be defined by the Dirac delta function
δ(t − τ ).

From the principle of impulse and momentum given in Equation 4.1, the impulse F̂ acting on a
body of mass m will result in sudden change in its velocity, without an appreciable change in its
displacement.

4.2.1 Impulse Response of Undamped Single Degree of Freedom Sys-


tem

Under free vibration, the response for the undamped 1-dof vibrating system with initial conditions
x(t = 0) = x0 and ẋ(t = 0) = ẋ0 was shown to be:
Vibrations: 4.2 Impulse Excitations 67

ẋ0
x(t) = x0 cos ωn t + sin ωn t (4.2)
ωn

If the mass is at rest before the unit impulse is applied, then at t < 0s, x = ẋ = 0. Therefore
using Equation 4.1 and noting that F̂ = f = 1, ẋ2 = ẋ(t = 0) = ẋ0 and ẋ1 = ẋ(t < 0) = 0, then
we can get the initial conditions of the system under unit impulse at t = 0 as:

x0 = 0
1 = m × ẋ(t = 0) − mẋ(t < 0) = mẋ0
1
ẋ0 =
m

Therefore the response of a undamped 1-dof system to the unit impulse which is also known as
impulse response function g(t), becomes:
1
x(t) = g(t) = sin ωn t (4.3)
mωn

4.2.2 Impulse Response of Under-damped Single Degree of Freedom


System

Under free vibration, the response for a under-damped 1-dof vibrating system with initial condi-
tions x(t = 0) = x0 and ẋ(t = 0) = ẋ0 was shown to be:

 ẋ0 + ζωn x0 
x(t) = e−ζωn t x0 cos ωd t + sin ωd t (4.4)
ωd
1
It has been shown that the initial conditions of system under unit impulse are x0 = 0 and ẋ0 = m
,
therefore the response of a under-damped 1-dof system to the unit impulse becomes:
e−ζωn t
x(t) = g(t) = sin ωd t (4.5)
mωd
Where:
r
k
ωn =
m
c
ζ =
2mωn
p
ωd = ωn 1 − ζ 2

The impulse response function g(t) on Equation 4.5 will be of form shown in Figure 4.2.

If the magnitude of the impulse is F̂ instead of unity, the initial velocity ẋ0 = m
is and the
response of the system becomes:
F̂ e−ζωn t
g(t) = sin ωd t (4.6)
mωd
Vibrations: 4.2 Impulse Excitations 68

Figure 4.2:

If the impulse F̂ is applied at an arbitrary time t = τ as shown in Figure 4.3(a), it will change

its velocity at t = τ by an amount m . Assuming that x=0 until the impulse is applied, the
displacement x at any subsequent time t, caused by a change in the velocity at time is given by
Equation 4.6 with t replaced by the time elapsed after the application of the impulse that is, t − τ .

Figure 4.3:
Vibrations: 4.3 Arbitrary Excitations 69

4.3 Arbitrary Excitations

Various methods can be used to find the response of a system under an arbitrary excitation. We
will consider the Convolution integral and the Laplace transformation methods.

4.3.1 Convolution Integral

Consider the response of the system under an arbitrary external force F(t), shown in Figure 4.4.
This force may be assumed to be made up of a series of impulses of varying magnitude.

Figure 4.4:

Assuming that at time τ , a force of F (τ ) acts on the system for a short duration of time, then the
impulse is:

F̂ (τ ) = F (τ )∆τ

At time t, the elapsed time after the impulse F̂ (τ ) is t − τ . The response of the system due to
this impulse alone is

x(t) = F (τ )∆τ g(t − τ )

The total response at time t can be found by summing (linear superposition) all the responses
due to the elementary impulses at all times, that is,
Z t
x(t) = F (τ )g(t − τ )dτ (4.7)
0

This integral is called the convolution or Duhamel integral.

Example 4.3.1. Determine the response of a single-DOF system to the step excitation shown in
Figure 4.5.
Vibrations: 4.3 Arbitrary Excitations 70

Figure 4.5: Step function excitation

Solution

Considering the undamped system, we have:


1
g(t) = sin ωn t (4.8)
mωn

By substituting into equation 4.7, the response of the undamped system is:
Z t
Fo
x(t) = sin ωn (t − τ )dτ
mωn 0
Fo h 1 i Fo h i
= (1 − cos ωn t) = 1 − cos ωn t
mωn ωn mωn2
Fo
= (1 − cos ωn t) (4.9)
k

Assignment 1
(a) A compacting machine, modeled as a single-degree-of-freedom system, is shown in Figure 6(a).
The force acting on the mass m (m includes the masses of the piston, the platform, and the
material being compacted) due to a sudden application of the pressure can be idealized as a
step force as shown in Figure 6(b). Show that, the response of the system is:

Fo h e−ζωn t p i
x = 1− p cos 1 − ζ 2 ωn t − ψ
k 1 − ζ2
ζ
where tan ψ = p
1 − ζ2
Vibrations: 4.3 Arbitrary Excitations 71

(a) (b)

Figure 4.6: (a) Compacting machine (b) Step force excitation

(b) If the compacting machine shown in Figure 6(a) is subjected to a force shown in Figure 4.7,
show that the response of the system now is:
Fo hp
2 −ζωn (t−t0 )
 i
x =
p 1−ζ −e cos ωd (t − t0 ) − ψ
1 − ζ2
k
ζ
where tan ψ = p
1 − ζ2

Figure 4.7: Step force applied with a time delay


Vibrations: 4.3 Arbitrary Excitations 72

Example 4.3.2. Time-delayed Step Force

Consider a one dof undamped system subjected to a time delayed step excitation force shown in
Figure 4.7. Determine the response of the system.

Solution

For a one dof undamped system, the impulse response function g(t), is:
1
x(t) = g(t) = sin ωn t
mωn

The convolution integral noting that the forcing function starts at t0 instead of t=0 becomes
Z t
Fo
x(t) = sin ωn (t − τ )dτ
mωn t0
Fo h 1 i Fo h i
= (1 − cos ωn (t − t0 )) = 1 − cos ωn (t − t0 )
mωn ωn mωn2
Fo  
= 1 − cos ωn (t − t0 )
k
Example 4.3.3. Linearly Varying Excitation Force (Ramp Force)

Consider a one dof undamped system subjected to a ramp excitation force shown in Figure 4.8.
Determine the response of the system.

Figure 4.8: Linear-varying force (Ramp force)

Solution

Let δF be the rate of increase of the force per unit time. Therefore, the forcing function at time
τ is:
F (τ ) = δF τ

Using the convolution integral the total response is:


Z t Z t
1 δF
x(t) = δF τ sin ωn (t − τ )dτ = τ sin ωn (t − τ )dτ
0 mωn mωn 0
Vibrations: 4.4 Description of a Transient Response 73

Note the following integral:


Z
x 1
x sin axdx = − cos ax + 2 sin ax
a a
Z
x 1
x cos axdx = sin ax + 2 cos ax
a a

We get:
δF h 1 τ it
x(t) = sin ωn (t − τ ) + cos ωn (t − τ )
mωn ωn2 ωn 0
δF 1 h i δF h i
= ωn t − sin ωn t = ω n t − sin ω n t
mωn ωn2 ωn k

Exercise

Mostly, the exciting ramp force has a rise time as shown in Figure 4.9. Show that the total
response of a single dof undamped system subjected to this type of excitation is:

F0 h sin ωn t sin ωn (t − t0 ) i
x(t) = 1− + for t > t0
k ωn t ωn t

Figure 4.9: Ramp force with rise time

4.4 Description of a Transient Response

The behavior of a vibrating mechanical system with transient response is described in terms of
parameters such as maximum overshoot, peak time, rise time, delay time, and settling time. These
parameters are shown in Figure 4.10, which denotes a typical step response of an under-damped
system.
Vibrations: 4.4 Description of a Transient Response 74

Figure 4.10: Description of transient response

(a) Peak time (tp ): The peak time is the time required for the response to attain the first peak of
the overshoot. The maximum amount the response overshoots occurs when the derivative of
x(t) is zero. For a step response of an under-damped system, the peak time can be shown to
be:
π
tp =
ωd

(b) Rise time (ts ): The rise time is the time needed for the response to rise from 10% to 90% of
the final or steady-state value for over-damped systems. For under-damped systems, usually,
the rise time is taken as the time required for the response to rise from 0% to 100% of the
final or steady-state value, and it can be shown to be:
π−α
tp =
ωd
where;
h ζω i h ζ i
−1 n −1
α = tan tan p
ωd 1 − ζ2

(c) Maximum overshoot (Mp ): The maximum overshoot is the maximum peak value of the re-
sponse compared to the final or steady-state value expressed as a percentage of the steady-state
value. For a step response of an under-damped system, the percentage overshoot can be shown
to be:
− √ ζπ
%Mp = 100e 1−ζ 2
Vibrations: 4.4 Description of a Transient Response 75

(d) Settling time (ts ): This is the time during which the response of the system x(t) reaches and
stays within ±2% of the steady-state or final value. For a step response of an under-damped
system, it can be shown to be:
 p 
− ln 0.02 1 − ζ 2
ts =
ζωn

(e) Delay time (td ): This is the time required for the response to reach 50% of the final or
steady-state value..
Chapter 5

Vibration of Continuous Systems

5.1 Introduction

In discrete (or lumped) vibrating systems, the mass, the damping and the elasticity are assumed
to be present only at certain discrete points in the system.

In many practical cases in vibration, it is not possible to identify these discrete masses, dampers
or springs. Hence these elements will be distributed throughout the system.

Such systems with distributed elements are known as continuous systems (or, systems with infinite
number of degrees of freedom since they are considered to be composed of infinite number of
displacement coordinates.

For discrete systems, the governing equations of motion are ordinary differential equations (ODEs),
while the governing equations of motion of a continuous system are partial differential equations
(PDEs) which are challenging to solve. However modeling a vibrating system as continuous rather
than as discrete provides more accurate information.

A continuous system will have infinite number of natural frequencies and normal modes. These
are obtained by applying initial and boundary conditions to the developed PDE.

5.2 Vibration Analysis of Continuous Systems Using An-


alytical Method

5.2.1 Longitudinal Vibration of Bars

Consider a thin uniform bar, subjected to axial forces as shown in Figure 5.1.

Due to axial forces, there will be displacements u along the length of the rod that will be a function
of both position x and time t.
Vibrations: 5.2 Vibration Analysis of Continuous Systems Using Analytical Method 77

Figure 5.1:

Consider an element on this rod of length dx. If u is the displacement at x, then displacement at
the other end of the element x + dx will be u + ∂u
∂x
dx.

The new length of the element is:


∂u ∂u
u + dx + dx − u = dx + dx
∂x ∂x

Therefore, the strain on the element is given by

Change in length dx + ∂u
∂x
dx − dx ∂u
ε= = = (5.1)
Initial length dx ∂x

From Hooke’s Law,


∂u P
ε= = (5.2)
∂x AE
Where A is the cross-sectional area of the bar which may be constant or varying longitudinally.

Assuming that A is constant, Equation 5.2 can be differentiated with respect to x to give,

∂P ∂ 2u
= AE 2 (5.3)
∂x ∂x
Applying Newton’s second law on the element,
X
F = mü
∂P ∂ 2u
P+ dx − P = m 2
∂x ∂t
∂P ∂ 2u
dx = ρAdx 2 (5.4)
∂x ∂t
where ρ is the density of the bar.
∂P
Substituting ∂x
from Equation 5.3, we obtain:

∂ 2u ∂ 2u
ρAdx = AE dx
∂t2 ∂x2
∂ 2u E ∂ 2u
⇒ 2 = (5.5)
∂t ρ ∂x2
Vibrations: 5.2 Vibration Analysis of Continuous Systems Using Analytical Method 78

q
Let c = Eρ = velocity of propagation of the displacement or stress wave in the rod. Equation
??, becomes:
∂ 2u 2
2∂ u
= c (5.6)
∂t2 ∂x2
which is a partial differential equation. One method of solving pde is that of separation of variables.
In this method,the solution is assumed to be in the form:

u(x, t) = U (x)G(t)

differentiating and substituting in equation 5.6, we obtain:

d2 G 2
2 d U
U = c G (5.7)
dt2 dx2
2
1dU 1 1 d2 G
or = (5.8)
U dx2 c2 G dt2
Because the LHS is independent of t and the RHS is independent of x, it follows that each side
2
must be a constant. Letting this constant be - ωc2 , we obtain 2 odes:

d2 U  ω  2
+ U = 0 (5.9)
dx2 c
d2 G
+ ω2G = 0 (5.10)
dt2
whose general solutions are:
ω ω
U (x) = A sin x + B cos x (5.11)
c c
G(t) = C sin ωt + D cos ωt (5.12)

The general solution to the pde becomes:


ω ω
u(x, t) = (A sin x + B cos x)(C sin ωt + D cos ωt) (5.13)
c c
The constants A, B, C and D are calculated from the boundary conditions and initial conditions:

Example 5.2.1. A bar of uniform cross-sectional area A, density ρ, modulus of elasticity E, and
length l is fixed at one end and free at the other end. It is subjected to an axial force at its free end,
as shown in Figure 2(a). Determine the natural frequencies and equation for the normal modes of
free longitudinal vibrations of the bar if F0 is suddenly removed.

Solution

The boundary conditions are:

u(0, t) = 0 ∵ No displacement at the fixed end


∂u(l, t)
= 0 ∵ No strain at the free end
∂x
Vibrations: 5.2 Vibration Analysis of Continuous Systems Using Analytical Method 79

(a) (b)

Figure 5.2:

Applying the boundary conditions,

0 = B(C sin ωt + D cos ωt) ⇒ B = 0


∂u ω ω
= A cos x(C sin ωt + D cos ωt)
∂x c c
ω ω
0 = A cos l(C sin ωt + D cos ωt)
c c
A 6= 0
6 0
C sin ωt + D cos ωt =
ω
⇒ cos l = 0 (5.14)
c
Equation 5.14 is called the frequency or characteristic equation and is satisfied by several values
of ω which are the natural frequencies of the problem.
ωl π 3π 5π
= , , .....
c 2 2 2

The n-th natural frequency is then given by:

ωn l (2n − 1)π
=
c 2
πc
ωn = (2n − 1) n = 1, 2, 3....
2l
The equation of response at the n-th vibration mode is:
ωn x
un (x, t) = A sin (C sin ωn t + D cos ωn t)
c
ωn x
= sin (Cn sin ωn t + Dn cos ωn t) (5.15)
c
Equation 5.15 is also called the n-th normal mode of vibration or n-th harmonic. The mode
corresponding to ωn = 1 is the fundamental mode and ωn = 1 is the fundamental frequency.

Constants Cn and Dn can be found from the initial conditions. Which are determined as follows:
Vibrations: 5.2 Vibration Analysis of Continuous Systems Using Analytical Method 80

(a) The displacement of the bar just before the force F0 is removed (initial displacement) is given
by:
F0 x
u(x, 0) = u0 = for 0 ≤ x ≤ l
EA
u(0, 0) = 0
F0 l
u(l, 0) =
EA
(b) Since the initial velocity is zero, we have;
∂u
u̇(x, 0) = u(x, 0) = 0 for 0 ≤ x ≤ l
∂t

The first time derivative of Equation 5.15 is:


ωn x
u̇(x, t) = sin (ωn Cn cos ωn t − ωn Dn sin ωn t) (5.16)
c

Applying the second initial condition u̇(x, 0) = 0 on Equation 5.16 leads to:
ωn x
0 = ωn Cn sin
c
⇒ Cn = 0

We then have:
ωn x
un (x, t) = Dn sin cos ωn t (5.17)
c

F0 l
Now applying the initial condition u(l, 0) = EA
on Equation 5.17 leads to:
F0 l ωn l
= Dn sin
EA c
F0 l
⇒ Dn =
EA sin ωcn l

Therefore, the solution for the normal modes of vibration of the bar becomes
F0 l ωn x
un (x, t) = sin cos ωn t (5.18)
EA sin ωcn l c

5.2.2 Torsional Vibration of Rods

Consider a circular shaft subjected to a torque as shown in Figure 5.3

The angle of twist of an element of length, dx can be obtained from the torsion equation:
T τ Gθ
= =
J r L
T ∂θ
⇒ = G
J ∂x
Vibrations: 5.2 Vibration Analysis of Continuous Systems Using Analytical Method 81

Figure 5.3: Torsional system

where J is the polar moment of inertia of the x-section. Integrating, we obtain:


∂T ∂ 2θ
= GJ 2 (5.19)
∂x ∂x
Equating the net torque to the inertia torque,
∂T ∂ 2θ
dx = Im 2 Im = ρLJ (5.20)
∂x ∂t
∂T
Replacing ∂x
from Equation 5.19, we obtain:
∂ 2θ ∂ 2θ
GJ = ρdxJ
∂x2 ∂t2
2 2
∂ θ G∂ θ
2
= (5.21)
∂t ρ ∂t2
This equation is the same as that of longitudinal vibration of rods. The general solution to this
equation is:
ω ω
θ(x, t) = (A sin x + B cos x)(C sin ωt + D cos ωt) (5.22)
c c
where: s
G
c=
ρ
Example 5.2.2. Determine the equations for the natural frequencies of a uniform shaft in tor-
sional oscillation when;

(a) one end fixed and the other free.

(b) both ends are fixed.

Determine also the fundamental frequency in each case if the shaft is of length 2m and the modulus
of rigidity and density of the shaft material are respectively 80GPa and 7800kg/m3 .
Vibrations: 5.2 Vibration Analysis of Continuous Systems Using Analytical Method 82

Solution

(a) For a fixed-free shaft, the boundary conditions are:

(i) At the fixed end, no angular displacements, that is; at x = 0, θ = 0


(ii) At the free end angular displacements are maximum, hence torque is zero, that is;at
∂θ
x = l, ∂x =0

0 = B(C sin ωt + D cos ωt) ⇒ B = 0


∂θ ω ω
= A cos x(C sin ωt + D cos ωt)
∂x c c
ω ω
0 = A cos l(C sin ωt + D cos ωt)
c c
ω ωl π 3π 5π
⇒ cos l = 0 or = , , ...
c c 2 2 2
The natural frequencies are then given by:
πc
ωn = (2n − 1) n = 1, 2, 3....
2l
The fundamental frequency is when n=1, hence
s
πc π G
ω1 = =
2l 2l ρ
r
π 80 × 109
= = 2515.285rad/s
2×2 7800

(b) For a fixed-fixed shaft, the boundary conditions are:

(i) At the fixed ends, no angular displacements, that is; at x = 0, θ = 0 and at x = l, θ = 0

0 = B(C sin ωt + D cos ωt) ⇒ B = 0


ω
0 = A sin l(C sin ωt + D cos ωt)
c
ω ωl
⇒ sin l = 0 or = π, 2π, 3π...
c c
The natural frequencies are then given by:
nπc
ωn = n = 1, 2, 3....
l
The fundamental frequency is when n=1, hence
s
πc π G
ω1 = =
l l ρ
r
π 80 × 109
= = 5030.57rad/s
2 7800
Vibrations: 5.2 Vibration Analysis of Continuous Systems Using Analytical Method 83

5.2.3 Lateral Vibrations of Beams

Consider a free-body diagram of an element on a beam shown in Figure 5.4 where M(x,t) is the
bending moment V(x,t) is the shear force and f(x,t) is the external force per unit length of the
beam. The beam has density ρ, second moment of area about the bending axis I, the Young’s
modulus E and area of the cross-section A. The element deflects by w(x, t) from the equilibrium
position.

Figure 5.4: A beam in bending

Using the Newton’s second law of motion, then balancing the forces vertically and moments about
axis OO’ on the element leads to the following partial differential equation:
∂ 4 w(x, t) ∂ 2 w(x, t)
EI + ρA = f (x, t) (5.23)
∂x4 ∂t2

For free lateral vibrations of the beam f(x,t)=0, hence Equation 5.23 reduces to:
∂ 4 w(x, t) ∂ 2 w(x, t)
c2 + = 0 (5.24)
∂x4 ∂t2
where
s
EI
c =
ρA

Solution for Free Lateral Vibrations of Beams

Equation 5.24 is a fourth order derivative of x and a second order derivative of t. Hence we need
four boundary conditions and two initial conditions to obtain a unique solution of the deflection
w(x, t). The equation can be solved by separation of variables.

w(x, t) = U (x)G(t)
4
∂ w(x, t) d4 U (x)
= G(t)
∂x4 dx4
2 2
∂ w(x, t) d G(t)
4
= U (x)
∂t dt2
Vibrations: 5.2 Vibration Analysis of Continuous Systems Using Analytical Method 84

Therefore:
d4 U (x) d2 G(t)
c2 G(t) + U (x) = 0
dx4 dt2
c2 d4 U (x) 1 d2 G(t)
4
= − 2
= constant = ω 2
U (x) dx G(t) dt

Lets solve for G(t):

d2 G(t)
2
+ ω 2 G(t) = 0
dt
This is a second order order whose solution is:

G(t) = A cos ωt + B sin ωt (5.25)

Lets solve for U (x):

d4 U (x)
c2 − U (x)ω 2 =
dx4
d4 U (x)
4
− β 2 U (x) = 0 (5.26)
dx
where
ω2 ω 2 ρA
β2 = =
c2 EI

The solution to Equation 5.26 is:

U (x) = C1 cos βx + C2 sin βx + C3 cosh βx + C4 sinh βx (5.27)

The general solution therefore is obtained by combining Equations 5.25 and 5.27 to get:
  
w(x, t) = C1 cos βx + C2 sin βx + C3 cosh βx + C4 sinh βx A cos ωt + B sin ωt (5.28)

Constants C1 , C2 , C3 and C4 are obtained from the boundary conditions. The natural frequencies
will be obtained once β is gotten, that is:
s s
EI EI
ω = β2 = (βl)2
ρA ρAl4
Vibrations: 5.2 Vibration Analysis of Continuous Systems Using Analytical Method 85

Common Boundary Conditions

(a) A free-end.

(i) At a free end of a beam, the bending moment is zero. That is:

∂ 2 w(x, t)
EI = 0
∂x2
∂ 2 w(x, t)
= 0
∂x2
dM
(ii) At a free end of a beam, the shear forces V = dx
are zero. That is:

∂ 3 w(x, t)
EI = 0
∂x3
∂ 3 w(x, t)
= 0
∂x3
(b) Pinned end.

(i) At a pinned end of a beam, the deflection of the beam is zero. That is:

w(x, t) = 0

(ii) At a pinned end of a beam, the bending moment is zero. That is:

∂ 2 w(x, t)
EI = 0
∂x2
∂ 2 w(x, t)
= 0
∂x2
(c) Fixed (or clamped) end.

(i) At a fixed end of a beam, the deflection of the beam is zero. That is:

w(x, t) = 0

(ii) At a fixed end of a beam, the slope of the deflection is zero. That is:

∂w(x, t)
= 0
∂x
Example 5.2.3. Determine the natural frequencies of vibrations of a uniform beam which is
rigidly fixed at x=0 and pinned at the other end.

Solution
Vibrations: 5.2 Vibration Analysis of Continuous Systems Using Analytical Method 86

The general solution which gives the normal mode or characteristic function of a beam undergoing
lateral vibrations is:
 
w(x, t) = C1 cos βx + C2 sin βx + C3 cosh βx + C4 sinh βx G(t)
∂w(x, t)  
= − βC1 sin βx + βC2 cos βx + βC3 sinh βx + βC4 cosh βx G(t)
∂x
2
∂ w(x, t) 
2 2 2 2

= − β C1 cos βx − β C2 sin βx + β C3 cosh βx + β C4 sinh βx G(t)
∂x2

All the four boundary conditions are:

w(0, t) = 0
∂w(0, t)
= 0
∂x
w(l, t) = 0
∂ 2 w(l, t)
= 0
∂x2

Applying the first boundary condition gives:

w(0, t) = (C1 + C3 )G(t) = 0


G(t) 6= 0
C1 + C3 = 0
C3 = −C1

Applying the second boundary condition leads to:


∂w(0, t)
= (C2 + C4 )G(t) = 0
∂x
G(t) 6= 0
C2 + C4 = 0
C4 = −C2

Deflection equation can now be written as:


 
w(x, t) = C1 (cos βx − cosh βx) + C2 (sin βx − sinh βx) G(t)

Now applying the third boundary condition leads to:


 
w(l, t) = C1 (cos βl − cosh βl) + C2 (sin βl − sinh βl) G(t) = 0
G(t) 6= 0
0 = C1 (cos βl − cosh βl) + C2 (sin βl − sinh βl) (5.29)
Vibrations: 5.2 Vibration Analysis of Continuous Systems Using Analytical Method 87

Applying the forth boundary condition leads to:

∂ 2 w(l, t) 2
 
= β − C 1 (cos βl + cosh βl) − C 2 (sin βl + sinh βl) G(t) = 0
∂x2
G(t) 6= 0
0 = −C1 (cos βl + cosh βl) − C2 (sin βl + sinh βl) (5.30)

Equations 5.29 and 5.30 can be written in matrix form as:


    
(cos βl − cosh βl) (sin βl − sinh βl) C1 0
= (5.31)
−(cos βl + cosh βl) (sin βl + sinh βl) C2 0

For non-trivial solution of C1 and C2 , then:

(cos βl − cosh βl) (sin βl − sinh βl)


= 0
−(cos βl + cosh βl) (sin βl + sinh βl)
(cos βl − cosh βl)(sin βl + sinh βl) + (sin βl − sinh βl)(cos βl + cosh βl) = 0 (5.32)

which when expanded and simplified leads to:

cos βl sinh βl = sin βl cosh βl


tan βl = tanh βl (5.33)

Equation 5.33 is the frequency equation and can be solved through trial and error for the values
of βl which satisfy this equations. These values of βl will give the natural frequencies of vibrations
of the beam.

Therefore: (βl)1 = 3.927, (βl)2 = 7.069, (βl)3 = 10.21 (radians). The fundamental natural
frequency is when n=1, that is for (βl)1 = 3.927 and can be shown to be:
s s s
EI EI 15.421 EI
ω1 = (βl)21 4
= 3.9272 4
= 2
rad/s
ρAl ρAl l ρA
s
ω1 1  15.421  EI
f1 = = Hz
2π 2π l2 ρA

Exercise

Show that:

(a) For a pinned-pinned beam of length l and made of isotropic material;


88
Vibrations: 5.3 Vibration Analysis of Continuous Systems Using Finite Element Method

(i) The frequency equation is given by,

sin βl = 0

(ii) The first three natural frequencies are;


s
0.5π EI
f1 = 2 Hz
l ρA
s
2π EI
f2 = 2 Hz
l ρA
s
4.5π EI
f3 = 2 Hz
l ρA

(b) For a cantilever beam of length l and made of isotropic material;

(i) The frequency equation is given by,

cos βl cosh βl = −1

(ii) The fundamental natural frequencies is;


s
0.559 EI
f1 = Hz
l2 ρA

5.3 Vibration Analysis of Continuous Systems Using Fi-


nite Element Method

5.3.1 Introduction

The finite element method is a numerical method that can be used for the accurate solution of
complex mechanical and structural vibration problems. It is a numerical procedure for solving
physical problems governed by a differential equation or an energy theorem. It has two charac-
teristics that distinguish it from other numerical procedures:

(a) The method utilizes an integral formulation to generate a system of algebraic equations.

(b) The method uses continuous piecewise smooth functions for approximating the unknown quan-
tity or quantities.

In this method, the actual structure is replaced by several pieces or elements, each of which is
assumed to behave as a continuous structural member called a finite element.
89
Vibrations: 5.3 Vibration Analysis of Continuous Systems Using Finite Element Method

The elements are assumed to be interconnected at certain points known as joints or nodes (Figure
5.5). Since it is very difficult to find the exact solution (such as the displacements) of the original
structure under the specified loads, a convenient approximate solution is assumed in each finite
element. The idea is that if the solutions of the various elements are selected properly, they can
be made to converge to the exact solution of the total structure as the element size is reduced.

Figure 5.5: Finite Element Model

During the solution process, the equilibrium of forces at the joints and the compatibility of dis-
placements between the elements are satisfied so that the entire structure (assemblage of elements)
is made to behave as a single entity

The application of the finite element method to vibration problems involves the following steps:

(a) Discretization of solution domain into a finite element mesh

(b) Development of element equations

(c) Assembly of element equations

(d) Solution for nodal unknowns

(e) Computation of solution and related quantities over each element

5.3.2 Mass and Stiffness Matrices for a Finite Element

Bar Element

Consider a bar element shown in Figure 5.6 which is extracted from a structure undergoing
longitudinal vibrations. The two ends form the nodes or joints.

Figure 5.6: Finite Element Model


90
Vibrations: 5.3 Vibration Analysis of Continuous Systems Using Finite Element Method

When the element is subjected to axial loads f1 (t) and f2 (t), the axial displacement within the
element assuming a linear shape function is:

u(x, t) = a(t) + b(t)x (5.34)

Applying boundary conditions:

at x = 0, u(0, t) = u1 (t) ⇒ a(t) = u1 (t)


at x = l, u(l, t) = u2 (t) ⇒ u2 (t) = u1 (t) + b(t)l
u2 (t) − u1 (t)
∴ b(t) =
l

Therefore, the shape function of the bar element becomes:

u2 (t) − u1 (t)  x x
u(x, t) = u1 (t) + x= 1− u1 (t) + u2 (t) (5.35)
l l l

The velocity function is:  x x


u̇(x, t) = 1 − u̇1 (t) + u̇2 (t) (5.36)
l l

The kinetic energy of the bar element is:

1 l  ∂u 2 1 l
Z Z
T (t) = ρA dx = ρAu̇2 dx
2 0 ∂t 2 0
1 l h
Z
x x i2
= ρA 1 − u̇1 (t) + u̇2 (t) dx
2 0 l l
2 3  x2
1 h x x x3  x3 i l
= ρA x − + 2 u̇21 + 2 − 2 u̇1 u̇2 + 2 u̇22
2 l 3l 2l 3l 3l 0
1 ρAl  2 
= u̇1 + u̇1 u̇2 + u̇22 (5.37)
2 3

The kinetic energy can also be expressed as:


1 T 1
T (t) = [u̇] [m] [u̇]
2 2  
1 h i u̇1
= u̇1 u̇2 [m]
2 u̇2
  
1 h i ρAl m11 m12 u̇1
= u̇1 u̇2 (5.38)
2 3 m21 m22 u̇2

Comparing this equation with Equation 5.37 leads to

1h i ρAl  1 1   u̇ 
2 1
T (t) = u̇1 u̇2 1
2 3 1 u̇ 2
 2  
1 h i ρAl 2 1 u̇1
= u̇1 u̇2 (5.39)
2 6 1 2 u̇2
91
Vibrations: 5.3 Vibration Analysis of Continuous Systems Using Finite Element Method

Therefore the mass matrix of a bar element is:


 
ρAl 2 1
[m] = (5.40)
6 1 2

The strain energy of the bar is:


1 l
Z  ∂u 2
V (t) = EA dx
2 0 ∂x
1 l
Z h u
1 u2 i2
= EA − + dx
2 0 l l
1 h u2 x 2u u x u2 x il
1 2
= EA 12 − + 22
2 l l2 l 0
1 EA  2 
= u1 − 2u1 u2 + u21 (5.41)
2 l

The strain energy can also be expressed as:


1 T 1
V (t) = [u] [k] [u]
2 2  
1h i u1
= u1 u2 [k]
2 u2
  
1 h i EA k11 k12 u1
= u1 u2 (5.42)
2 l k21 k22 u2

Comparing this equation with Equation 5.41 leads to


1h i EA  1 −1   u 
1
T (t) = u1 u2 (5.43)
2 l −1 1 u2
Therefore the stiffness matrix of a bar element is:
 
EA 1 −1
[k] = (5.44)
l −1 1

Torsion Element

Similarly, the mass and stiffness matrices of an element from a structure undergoing torsional
vibrations can be shown to be:
 
ρJl 2 1
[m] = (5.45)
6 1 2
and
 
GJ 1 −1
[k] = (5.46)
l −1 1

Where:

J is the polar moment of area of the cross-section about the centroidal axes

G is the modulus of rigidity of the material of the structure


92
Vibrations: 5.3 Vibration Analysis of Continuous Systems Using Finite Element Method

5.3.3 Equation of Motion of the Complete System of Finite Elements

A complete vibrating structure is considered an assemblage of several finite elements.

Therefore, the equations of motion obtained for single finite elements should be combined to get
an equation of motion representing the complete structure.

To illustrate the procedure, lets consider Figure 5.5 which shows a structure modeled with four
finite elements. The displacements of the joints are u1 , u2 , u3 , u4 and u5 which can be represented
as a column vector as:
 
u1
 u2 
 
[ū] =  u3 

 u4 
u5

The displacements of each element are:


  
1 0 0 0 0 u1
  0 1 0 0 0   u2 
 
u1  
[ū1 ] = 0
= 0 0 0 0  u3 
 
u2 0 0 0 0 0   u4 
0 0 0 0 0 u5

  
0 0 0 0 0 u1
  0 1 0 0 0   u2 
 
u2  
[ū2 ] = 0
= 0 1 0 0  u3 
 
u3 0 0 0 0 0   u4 
0 0 0 0 0 u5

  
0 0 0 0 0 u1
  0 0 0 0 0   u2 
 
u3  
[ū3 ] = 0
= 0 1 0 0  u3 
 
u4 0 0 0 1 0   u4 
0 0 0 0 0 u5

  
0 0 0 0 0 u1
  0 0 0 0 0  u2 
u4   
[ū4 ] = 0
= 0 0 0 0  u3 
 
u5 0 0 0 1 0   u4 
0 0 0 0 1 u5

It is seen that the joint displacement of any element (e) can be identified in the vector of node
displacement of the complete structure. Therefore the vector [ūe ] and [ū] are related as:

[ūe ] = Ae [ū]
93
Vibrations: 5.3 Vibration Analysis of Continuous Systems Using Finite Element Method

where Ae is a rectangular matrix for element e composed of zeros and ones.

The overall mass and stiffness matrices of a structure modeled with N-elements can be shown to
be:
N h
X i
[M] = Ae T [me ]Ae
e=1

and
N h
X i
T
[K] = Ae [ke ]Ae
e=1

where [me ] and [ke ] are the mass and stiffness matrices of element e.

5.3.4 Incorporation of Boundary Conditions

In our analysis, it has been assumed that no joint is fixed, and hence the structure is capable of
undergoing a rigid-body motion under the joint forces.

Usually structures are supported such that displacements are zero at a number of joints to avoid
rigid-body motion of the structure.

A simple method of incorporating the zero-displacement conditions is to eliminate the correspond-


ing rows and columns from the overall matrices [M] and [K]

Example 5.3.1. Determine the equation of motion and the natural frequencies for the longitudinal
vibration of a 2-section bar shown in Figure 5.7 by using two finite elements.

Figure 5.7: 2-section bar

Solution

The mass and stiffness matrices of element 1 are:


   
ρ(2A)l 2 1 ρAl 4 2
m1 = =
6 1 2 6 2 4
   
E(2A) 1 −1 EA 2 −2
k1 = =
l −1 1 l −2 2
94
Vibrations: 5.3 Vibration Analysis of Continuous Systems Using Finite Element Method

The mass and stiffness matrices of element 2 are:


 
ρAl 2 1
m2 =
6 1 2
 
EA) 1 −1
k2 =
l −1 1

Matrices A1 and A2 are:


 
    u1
u1 1 0 0  u2 
u1 = =
u2 0 1 0
u3
 
1 0 0
⇒ A1 =
0 1 0
 
    u1
u2 0 1 0  
u2 = = u2
u3 0 0 1
u3
 
0 1 0
⇒ A2 =
0 0 1

The overall mass and stiffness matrices of the bar are:

M = AT m A + AT 2 m2 A2
1 1 1  
1 0    0 0   
ρAl 4 2 1 0 0 ρAl 2 1 0 1 0
=  0 1  + 1 0
 
6 2 4 0 1 0 6 1 2 0 0 1
0 0 0 1
 
4 2 0
ρAl 
= 2 6 1
6
0 1 2
and

K = AT k A + AT2 k 2 A2
1 1 1  
1 0    0 0   
EA 2 −2 1 0 0 EA 1 −1 0 1 0
=  0 1  + 1 0
 
l −2 2 0 1 0 l −1 1 0 0 1
0 0 0 1
 
2 −2 0
EA 
= −2 3 −1 
l
0 −1 1

Therefore the equation of motion becomes:

   Mü
 + Ku
  = 
0 
4 2 0 ü1 2 −2 0 u1 0
ρAl  EA 
2 6 1   ü2 +
 −2 3 −1   u2  =  0
6 l
0 1 2 ü3 0 −1 1 u3 0
95
Vibrations: 5.3 Vibration Analysis of Continuous Systems Using Finite Element Method

Joint 1 is stationary, and hence its displacement is zero. Lets incorporate this boundary condition
by deleting the first row and column of the matrices in the equation of motion. This now gives:
       
ρAl 6 1 ü2 EA 3 −1 u2 0
+ = (5.47)
6 1 2 ü3 l −1 1 u3 0

Equation 5.47 is a 2dof equation of motion which can be solved using methods discussed in earlier.

Lets solve it for the eigenvalues (natural frequencies) by setting the determinant of the character-
istic matrix to zero.

The dynamic matrix is:


 −1    
−1 6 6 1 EA 3 −1 6E 0.6364 −0.2727
A = M K= = 2 (5.48)
ρAl 1 2 l −1 1 ρl 0.8182 0.6364

Therefore:

[A − ω 2 I]U = 0

h 6E 0.6364 −0.2727  
    
2 1 0 i U2 0
−ω =
ρl 2 0.8182 0.6364 0 1 U 3 0
ω 2 ρl2 1 0 i U2

h 0.6364 −0.2727       
0
− =
0.8182 0.6364 6E 0 1 U3 0
h  0.6364 −0.2727  
1 0
i
U2
  
0
−λ =
0.8182 0.6364 0 1 U3 0
Therefore:
   
0.6364 −0.2727 1 0
−λ = 0
0.8182 0.6364 0 1
 
0.6364 − λ −0.2727
= 0
0.8182 0.6364 − λ
which when expanded and solved gives:

λ1 = 0.164
λ2 = 1.1087

which gives the natural frequencies as:


√ s s
6λ1 E 0.992 E
ω1 = =
l ρ l ρ
√ s s
6λ2 E 2.579 E
ω2 = =
l ρ l ρ
96
Vibrations: 5.3 Vibration Analysis of Continuous Systems Using Finite Element Method

Assignment 2

Figure 5.8 shows a stepped marine engine shaft connected to a flywheel. Since the inertia of the
flywheel is relatively large, the flywheel can be assumed to be grounded. Determine the natural
frequencies in torsional vibration by modeling the shaft with three finite elements. The shaft is
made from an isotropic material with a density ρ and modulus of rigidity G.

Figure 5.8:
Chapter 6

Vibration Measurement

In some practical situations, it might be difficult to develop a mathematical model of the system
and predict its vibration characteristics through an analytical study. In such cases,experimental
methods are used to measure the vibration response of the system to a known input.

Vibration measurement is necessary for the following reasons:

(a) Maintenance of machines. The periodic measurement of vibration characteristics of a machin-


ery or structure is essential for ensuring adequate safety margins. Any observed shift in the
natural frequencies or other vibration characteristics will indicate either a failure or a need
for maintenance of the machine.

(b) The measurement of the natural frequencies of a structure or machine is useful in selecting
the operational speeds to avoid resonant conditions.

(c) To identify the mass, stiffness and damping elements of a vibrating system for use in a vibration
model.

(d) The theoretically computed vibration characteristics of a machine or structure may be different
from the actual values due to the assumptions made in the analysis.

(e) To validate the developed vibrational models of a structure or machine.

6.1 Vibration Measurement Scheme

Figure 6.1 shows the elements of a vibration measurement scheme.

• The motion of the vibrating body is converted into an electrical signal by the vibrating
transducer or pickup. In general, a transducer is a device that transforms changes in me-
chanical quantities (such as displacement, velocity, acceleration, or force) into changes in
electrical quantities (such as voltage or current).
Vibrations: 6.2 Transducers 98

Figure 6.1: Elements of a vibration measurement scheme

• Since the output signal (voltage or current) of a transducer is too small to be recorded
directly, a signal conversion instrument is used to amplify the signal to the required value.

• The output from the signal conversion instrument can be displayed on a display unit for
visual inspection, or recorded by a recording unit or stored in a computer for later use.

• The data can then be analyzed to determine the desired vibration characteristics of the
machine or structure.

Depending on the quantity been measured, a vibration measuring instrument is called a vibrom-
eter, a velocity meter (velometer), an accelerometer, a phase meter, or a frequency meter. The
following are factors to consider when selecting the type of vibration measuring instrument to be
used:

• Expected ranges of the frequencies and amplitudes.

• Sizes of the machine/structure involved.

• Conditions of operation of the machine/equipment/structure,

• Type of data processing used (such as graphical display or graphical recording or storing the
record in digital form for computer processing).

6.2 Transducers

A transducer is a device that transforms changes of physical/mechanical variables (such as dis-


placement, velocity, acceleration, or force) into equivalent electrical signals. The size of a trans-
ducer should not affect the dynamics of the structure of machine through added mass or stiffness.

Some of the transducers commonly used for vibration measurement are:


Vibrations: 6.2 Transducers 99

6.2.1 Variable Resistance Transducers

These are the most commonly used types of transducers in which a mechanical motion produces
a change in electrical resistance (of a rheostat, a strain gage, or a semiconductor), which in turn
causes a change in the output voltage or current.

They work on the principle that, the resistance of a conductor is directly proportional to length
(L) of the conductor and inversely proportional to the area (A) of the conductor. That is:

ρA
R =
L
Where ρ is the resistivity of a material measured in ohm-m.

6.2.2 Piezoelectric Transducers

Certain natural and manufactured materials like quartz, tourmaline, lithium sulfate, and Rochelle
salt generate electrical charge when subjected to a deformation or mechanical stress. The electrical
charge disappears when the mechanical loading is removed. Such materials are called piezoelectric
materials.

A piezoelectric transducer uses the piezoelectric effect to measure pressure, acceleration, strain or
force by converting them to an electrical charge.

A typical piezoelectric transducer is shown in Figure 6.2. A small mass is spring loaded against a
piezoelectric crystal (especially of quartz). When the base vibrates, the load exerted by the mass
on the crystal changes with acceleration, hence the output voltage generated by the crystal will
be proportional to the acceleration.

Figure 6.2:

The main advantages of the piezoelectric transducers include compactness, ruggedness, high sensi-
tivity, and high frequency range. They are the mostly used transducers in vibration measurement.
Vibrations: 6.3 Vibration Pickups 100

6.2.3 Linear Variable Differential Transformer (LVDT) Transducer

LVDT can be used to measure linear displacement or position of a vibrating body.

As shown in Figure 6.3, the LVDT consists of a primary coil at the center and two secondary
coils at the ends. A cylindrical ferromagnetic core, attached to the object whose position is to
be measured, slides freely inside the coils in the axial direction. This induces voltage differences
between the secondary coils proportional to the distance moved by the core..

Figure 6.3:

6.3 Vibration Pickups

When a transducer is used in conjunction with another device to measure vibrations, it is called
a vibration pickup. Vibration pickups are used to measure the displacement, velocity and accel-
eration of a vibrating body.

The vibration pickups for measuring displacement, velocity and acceleration of a vibrating body
are respectively referred to as vibrometers, velometers and accelerometers.

The commonly used vibration pickups are known as seismic instruments in which the vibrating
motion is measured relative to the mass of a mass-spring-damper system attached to the vibrating
structure.

As shown in Figure 6.4, the seismic instrument consists of:

(a) the frame which is rigidly attached to the vibrating structure,

(b) a mass supported on a linear spring and damper which are fastened to the frame of the
instrument,

(c) the transducer that measures the relative motion of the mass and the frame. The transducer
converts the measured mechanical motion to an electrical signal.
Vibrations: 6.3 Vibration Pickups 101

Figure 6.4:

With this arrangement, the bottom ends of the spring and the dashpot will have the same motion
as the frame and their vibration excites the suspended mass into motion.

Assuming the vibrating body has a harmonic motion, then:

y(t) = Y sin ωt (6.1)

The relative motion is given by:

z = x−y
ż = ẋ − ẏ
z̈ = ẍ − ÿ

The equation of motion of the mass is given by:

mẍ + c(ẋ − ẏ) + k(x − y) = 0


m(z̈ + ÿ) + cż + kz = 0
mz̈ + cż + kz = −mÿ
mz̈ + cż + kz = mω 2 Y sin ωt (6.2)

Equation 6.2 represents a forced vibration in which Fo = mω 2 Y . The steady state solution to this
equation can be represented as:
z = Z sin(ωt − φ) (6.3)

where Z and φ are given by:


mY ω 2 r2 Y
Z = p =p (6.4)
[(k − mω 2 )2 + c2 ω 2 ] [(1 − r2 )2 + (2ζr)2 ]
cω 2ζr
φ = tan−1 2
=
k − mω 1 − r2
ω
r =
ωn

Z ω
The variation of Y
with respect to r = ωn
for various values of ζ is shown in Figure 6.5.
Vibrations: 6.3 Vibration Pickups 102

Figure 6.5:

6.3.1 Vibrometer

This is seismic instrument used to measure the displacement of a vibrating body.

It operates on the range when ωωn ≥ 3 where as shown in Figure 6.5 Z ' Y , that is, the relative
displacement of the mass (Z) measured by the transducer is approximately the same as the the
displacement of the vibrating body (Y). Therefore the recorded displacement is:

z(t) ' Y sin(ωt − φ)

if:
Z r2
=p ' 1
Y [(1 − r2 )2 + (2ζr)2 ]

However, the recorded displacement z(t) lags behind the displacement being measured y(t) by
time t0 = ωφ .

Since the vibrometer operates on a large ratio of ωωn and the frequency ω of the vibrating machine
q
k
is fixed, then the natural frequency ωn = m of the vibrometer must be low. This means that the
vibrometer should have a large mass and a low spring stiffness. This results in a bulky instrument,
which is not desirable in many applications.

6.3.2 Accelerometers

An accelerometer is an instrument that measures the acceleration of a vibrating body and are
commonly used for vibration measurement. From the accelerometer record, the velocity and
displacements are obtained by integration.
Vibrations: 6.3 Vibration Pickups 103

Figure 6.6:

Differentiating Equation 6.3, gives:

z̈(t) = −ω 2 Z sin(ωt − φ) (6.5)


z̈(t)
⇒ 2 = −Z sin(ωt − φ) = −z(t) (6.6)
ω
Substituting the value of Z as in Equation 6.4 to Equation 6.5 leads to:
r2  
z̈(t) = p − ω 2 Y sin(ωt − φ)
[(1 − r2 )2 + (2ζr)2 ]
z̈(t) 1 
2

= p − ω Y sin(ωt − φ)
r2 [(1 − r2 )2 + (2ζr)2 ]
z̈(t)ωn2 1 
2

= p − ω Y sin(ωt − φ)
ω2 [(1 − r2 )2 + (2ζr)2 ]
2 1 
2

−z(t)ωn = p − ω Y sin(ωt − φ) (6.7)
[(1 − r2 )2 + (2ζr)2 ]
Equation 6.7 shows that if:
1
p ' 1 (6.8)
[(1 − r2 )2 + (2ζr)2 ]
then;

− z(t)ωn2 ' −ω 2 Y sin(ωt − φ) (6.9)

By comparing Equation 6.9 with ÿ(t) = −ω 2 Y sin ω the term z(t)ωn2 gives the acceleration ÿ(t)
of the vibrating body but with a phase lag φ. The time by which the recorded value lags the
acceleration is given by t0 = ωφ .
Vibrations: 6.3 Vibration Pickups 104

ω
It can be shown that the accelerometers operates at a range when < 1 Since the frequency ω of
ωn
q
k
the vibrating machine is fixed, then the natural frequency of the ωn = m of the accelerometer
must be high. This means that the accelerometer should have a small mass and a high spring
stiffness. This results to a small instrument in size. Due to their small size and high sensitivity,
accelerometers are preferred in vibration measurements.

Practically, Equation 6.8 may not be satisfied exactly and in such cases the quantity:
1 measured value
p = (6.10)
[(1 − r2 )2 + (2ζr)2 ] actual value

is used to find the correct value of the acceleration measured. This is illustrated in the following
example.

Example 6.3.1. An accelerometer has a suspended mass of 0.01 kg with a damped natural fre-
quency of vibration of 150 Hz. When mounted on an engine undergoing an acceleration of 1 g at
an operating speed of 6000 rpm, the acceleration is recorded as 9.5 m/s2 by the instrument. Find
the damping constant and the spring stiffness of the accelerometer.

Solution

The ratio of the measured acceleration to true acceleration is given by:

1 −z(t)ωn2 measured value 9.5


p = 2
= =
(1 − r2 )2 + (2ζr)2 −Y ω sin(ωt − φ) true value 9.81

(1 − r2 )2 + (2ζr)2 = 1.0663 (6.11)

The operating speed of the engine is:


2πN 2π × 6000
ω= = = 628.32 rad/s
60 60

The damped natural frequency of the accelerometer is:


p
ωd = ωn 1 − ζ 2 = 2π × 150 = 942.48 rad/s
ω ω r 628.32
⇒ = p =√ = = 0.667
ωd ωn 1 − ζ 2 1−r 2 942.48
∴ r2 = 0.444(1 − ζ 2 )

Substituting r2 in equation 6.11 and simplifying gives:

1.579ζ 4 − 2.27ζ 2 + 0.7573 = 0


⇒ ζ = 0.725, 0.911
Vibrations: 6.4 Signal Analysis 105

Taking ζ = 0.725 (to avoid the vicinity of critically damped vibrations),

ωd
ωn = p = 1368.4 rad/s
1 − ζ2
r
k
ωn = ⇒ k = mωn2 = 18, 725 N/m
m
c = 2mωn ζ = 19.84 Ns/m

6.4 Signal Analysis

Once the vibration characteristics of a system are measured using a vibration pickup, analysis of
the produced signals should therefore done to derive useful and inherent information regarding
the vibrations.

Vibration pickups (accceloremeters, vibrometers and velometers) produce analogue signals which
can be directly fed to a computer using data acquisition cards for display, analysis and storage of
the recorded response of the vibrating system. These signals however, give the time response of
the system which often will not provide much useful information of the vibrations.

Therefore, transformation of the time-domain signals produced by vibration pickups into frequency-
domain signals is mostly required in order to obtain useful information regarding the vibrations,
such as to identify possible causes of the vibrations/excitations.

The frequency response can show one or more discrete frequencies around which the energy is con-
centrated, hence help identifying the cause of vibration/excitation. For example, the acceleration-
time history of a machine frame that is subjected to excessive vibration might appear as shown
in Figure 6.7(a). This figure cannot be used to identify the cause of vibration. If the acceleration-
time history is transformed into the frequency-domain, the resulting frequency spectrum might
appear as shown in Figure 6.7(b), where the energy is shown to be concentrated around 25Hz.
This frequency can easily be related, for example, to the rotational speed of a particular motor.
Changing either this motor or its speed of operation, might avoid resonance and hence solve the
problem of excessive vibrations.
Vibrations: 6.4 Signal Analysis 106

Figure 6.7:

6.4.1 Use of Fast Fourier Transform in Signal Analysis

Digital filtering method and Fast Fourier Transform (FFT) are two types of real-time procedures
used to transform time-domain signals to frequency-domain signals.

The FFT procedure is the most commonly used method. Most modern vibration measurement
and analysis equipment are fitted with an FFT spectrum (frequency) analyzer.

Spectrum analyzers utilizing FFT converts the analog time-domain signals, into digital frequency-
domain data using Fourier series relations. This is done by separating the energy of the input
time-domain signal into various frequency bands using a set of filters.

Potrebbero piacerti anche